Sunteți pe pagina 1din 94

Set 3 Block 2:

Exam Title :
Economics & ...
Email : misrapulkit@yahoo.in
Contact :

QUESTION 1.
Consider the following statements

1. Stocks are defined over a period of time.

2. Flows are defined at a particular point of time.

Select the correct statement/s

a) 1 only
b) 2 only
c) Both
d) None
Correct Answer: D
Your Answer: Unanswered
Explanation

Solution (d)

None of the above are correct.

Flows are defined over a period of time . Often we hear statements like the average salary
of someone is Rs 10,000 or the output of the steel industry is so many tonnes or so many rupees
in value. But these are incomplete statements because it is not clear whether the income which
is being referred to is yearly or monthly or daily income and surely that makes a huge
difference. Sometimes, when the context is familiar, we assume that the time period is known
and therefore do not mention it. But inherent in all such statements is a definite period of time.
Otherwise such statements are meaningless. Thus income, or output, or profits are concepts
that make sense only when a time period is specified. These are called flows because they occur
in a period of time. Flow refers to the total value of transactions (sales or purchases, incomes or
expenditures) during an accounting period

IASbaba
Web: http://ilp.iasbaba.com/ Score:
Email: ilp@iasbaba.com 0.00 / 160
Page 1
Set 3 Block 2:
Exam Title :
Economics & ...
Email : misrapulkit@yahoo.in
Contact :

Stocks are defined at a particular point of time . capital goods or consumer durables once
produced do not wear out or get consumed in a delineated time period. In fact capital goods
continue to serve us through different cycles of production. The buildings or machines in a
factory are there irrespective of the specific time period. There can be addition to, or deduction
from, these if a new machine is added or a machine falls in disuse and is not replaced. A
particular machine can be part of the capital stock for many years (unless it wears out); but that
machine can be part of the flow of new machines added to the capital stock only for a single
year when it was initially installed. Stock refers to the value of an asset at a balance date (or
point in time).

Example:

Suppose a tank is being filled with water coming from a tap. The amount of water which is
flowing into the tank from the tap per minute is a flow. But how much water there is in the tank
at a particular point of time is a stock concept.

THINK!

• Wealth
• System Dynamics

Source : NCERT Class 12th- Introductory Macro Economics

IASbaba
Web: http://ilp.iasbaba.com/ Score:
Email: ilp@iasbaba.com 0.00 / 160
Page 2
Set 3 Block 2:
Exam Title :
Economics & ...
Email : misrapulkit@yahoo.in
Contact :
QUESTION 2.
Who is considered as founding father of modern economics?

a) Adam Smith
b) Mahalanobis
c) J.M. Keynes
d) Gopal Krishna Sarangi
Correct Answer: A
Your Answer: Unanswered
Explanation

Solution (a)

Adam smith is the answer

Adam Smith is regarded as the founding father of modern economics (it was known as political
economy at that time). He was a Scotsman and a professor at the University of Glasgow.

Philosopher by training, his well-known work 'An Enquiry into the Nature and Cause of the
Wealth of Nations' (1776) is regarded as the first major comprehensive book on the subject. The
passage from the book. ‘It is not from the benevolence of the butcher, the brewer, of the baker,
that we expect our dinner, but from their regard to their own interest. We address ourselves,
not to their humanity but to their self-love, and never talk to them of our own necessities but of
their advantage’ is often cited as an advocacy for free market economy. The Physiocrats of
France were prominent thinkers of political economy before Smith.

John Maynard Keynes(J.M.Keynes) was a British economist and was born in 1883.

He was educated in King’s College, Cambridge, United Kingdom and later appointed its Dean.
Apart from being a sharp intellectual he actively involved in international diplomacy during the
years following the First World War. He prophesied the breakdown of the peace agreement of
the War in the book The Economic Consequences of the Peace (1919). His book General Theory
of Employment, Interest and Money (1936) is regarded as one of the most influential economics
books of the twentieth century. He was also a shrewd foreign currency speculator.

Mahalanobis was an Indian scientist and applied statistician. He is best remembered for the M
ahalanobis distance , a statistical measure and for being one of the members of the first Plannin
g Commission of free India. He made pioneering studies in anthropometry in India. He founded
the Indian Statistical Institute , and contributed to the design of large-scale sample surveys.

Gopal Krishna Sarangi is an Indian economist specializing in energy economics , climate


change , energy regulations, energy access, etc. He is at present an Assistant Professor at
Department of Policy Studies, Teri University . His doctoral thesis at Teri University was
entitled, "Electricity sector regulation and sustainable development outcomes: An analysis of
regulatory impact in 12 Indian states for 2001–2010." under the supervision of Professor
Arabinda Mishra .

THINK!

• Economic Thinkers of Modern India

Source : NCERT Class 12th - Introductory Macro Economics

IASbaba
Web: http://ilp.iasbaba.com/ Score:
Email: ilp@iasbaba.com 0.00 / 160
Page 3
Set 3 Block 2:
Exam Title :
Economics & ...
Email : misrapulkit@yahoo.in
Contact :

QUESTION 3.
Consider the following factors of production and their remuneration

1. Human Labour- Wage


2. Capital - Interest
3. Entrepreneurship- Profit
4. Natural Resources- Rent

Choose the correct match

a) 1,2 and 4
b) 1,3 and 4
c) All of the above
d) None of the above
Correct Answer: C
Your Answer: Unanswered
Explanation

Solution (c)

There may fundamentally be four kinds of contributions that can be made during the production
of goods and services.

1. Contribution made by human labour, remuneration for which is called wage.


2. Contribution made by capital, remuneration for which is called interest.
3. Contribution made by entrepreneurship, remuneration of which is profit.
4. Contribution made by fixed natural resources (called ‘land’), remuneration for which is
called rent.

IASbaba
Web: http://ilp.iasbaba.com/ Score:
Email: ilp@iasbaba.com 0.00 / 160
Page 4
Set 3 Block 2:
Exam Title :
Economics & ...
Email : misrapulkit@yahoo.in
Contact :

Think!

• Circular flow of income


• methods of calculating national income

Source : NCERT Class 12th - Introductory Macro Economics

QUESTION 4.
Consider the following statements

1. Measuring the aggregate values of spending that the firms receive for the final goods and
services which they produce is called Expenditure method.
2. Method of measuring the aggregate values of final goods and services produced by all the
firms is called Product method.
3. Method of measuring the sum total of all factor payments are called Income method

Choose the INCORRECT statement/s

a) 1 and 2 only
b) 2 and 3 only
c) 1 and 3 only
d) None of the above
Correct Answer: D
Your Answer: Unanswered
Explanation

Solution (d)

All the statements are correct.

Measuring the aggregate values of spending, that the firms receive for the final goods and
services which they produce is called Expenditure method.

Method of measuring the aggregate values of final goods and services produced by all the firms
is called Product method.

Method of measuring the sum total of all factor payments are called Income method.

THINK!

• GDP, GNP, NNP.


• Gross national happiness (GNH)

Source : NCERT Class 12th- Introductory Macro Economics

QUESTION 5.
Gross National Product is calculated

IASbaba
Web: http://ilp.iasbaba.com/ Score:
Email: ilp@iasbaba.com 0.00 / 160
Page 5
Set 3 Block 2:
Exam Title :
Economics & ...
Email : misrapulkit@yahoo.in
Contact :

a) By measuring the sum total of gross value added by all the firms in the economy including
depreciation.
b) By measuring the sum total of gross value added by all the firms in the economy excluding
depreciation.
c) By measuring the sum total of gross value added by all the firms in the economy including
depreciation and net factor income from abroad.
d) By measuring the sum total of gross value added by all the firms in the economy excluding
depreciation and net factor income from abroad
Correct Answer: C
Your Answer: Unanswered
Explanation

Solution (c)

The macroeconomic variable which takes into account such additions and subtractions is known
as Gross National Product (GNP).

GNP measures the total monetary value of the total output produced by a country's residents.
Therefore, any output produced by foreign residents within the country's borders must be
excluded in calculations of GNP, while any output produced by the country's residents outside of
its borders must be counted.

GNP is related to another important economic measure called gross domestic product (GDP),
which takes into account all output produced within a country's borders regardless of who owns
the means of production. GNP starts with GDP, adds residents' investment income from
overseas investments, and subtracts foreign residents' investment income earned within a
country.

GNP = GDP + Factor income earned by the domestic factors of production employed in the rest
of the world – Factor income earned by the factors of production of the rest of the world
employed in the domestic economy.

Hence, GNP = GDP + Net factor income from abroad

(Net factor income from abroad = Factor income earned by the domestic factors of production
employed in the rest of the world – Factor income earned by the factors of production of the
rest of the world employed in the domestic economy).

Do you know?

The United States used GNP as its primary measure of total economic activity until 1991 after
which it began to use GDP.

THINK!

• Difference between GDP,GNP and NNP.


• Green National Product
• Gross National Income.

Source : NCERT Class 12th - Introductory Macro Economics

IASbaba
Web: http://ilp.iasbaba.com/ Score:
Email: ilp@iasbaba.com 0.00 / 160
Page 6
Set 3 Block 2:
Exam Title :
Economics & ...
Email : misrapulkit@yahoo.in
Contact :

QUESTION 6.
Consider the following definitions

1. Real GDP is the value of GDP evaluated at current market prices.

2. Nominal GDP is the value of GDP evaluated at constant prices.

3. GDP deflator is a ratio of real GDP to nominal GDP.

Choose the correct statement/s

a) 1 and 2 only
b) 2 and 3 only
c) All of the above
d) None of the above
Correct Answer: D
Your Answer: Unanswered
Explanation

Solution (d)

In order to compare the GDP figures (and other macroeconomic variables) of different countries
or to compare the GDP figures of the same country at different points of time, we cannot rely on
GDPs evaluated at current market prices. For comparison we take the help of real GDP.

Real GDP is calculated in a way such that the goods and services are evaluated at some
constant set of prices (or constant prices). Since these prices remain fixed, if the Real GDP
changes we can be sure that it is the volume of production which is undergoing changes. Real
GDP is a macroeconomic measure of the value of economic output adjusted for price changes
(i.e., inflation or deflation ).

Nominal GDP , on the other hand, is simply the value of GDP at the current prevailing prices.

For example, suppose a country only produces bread. In the year 2000 it had produced 100
units of bread, price was Rs 10 per bread.GDP at current price was Rs 1,000. In 2001 the same
country produced 110 units of bread at price Rs 15 per bread. Therefore nominal GDP in 2001
was Rs 1,650 (=110 × Rs 15). Real GDP in 2001 calculated at the price of the year 2000 (2000
will be called the base year) will be 110 × Rs 10 = Rs 1,100

The ratio of nominal GDP to real GDP gives us an idea of how the prices have moved from the
base year (the year whose prices are being used to calculate the real GDP) to the current year.
In the calculation of real and nominal GDP of the current year, the volume of production is
fixed. Therefore, if these measures differ it is only due to change in the price level between the
base year and the current year.

GDP Deflator(implicit price deflator) is the ratio of nominal to real GDP .Thus if GDP stands
for nominal GDP and gdp stands for real GDP then, GDP deflator =GDP/gdp. Like the consumer
price index (CPI), the GDP deflator is a measure of price inflation/deflation with respect to a
specific base year; the GDP deflator of the base year itself is equal to 100. Unlike the CPI, the
GDP deflator is not based on a fixed basket of goods and services; the "basket" for the GDP
deflator is allowed to change from year to year with people's consumption and investment
patterns.

IASbaba
Web: http://ilp.iasbaba.com/ Score:
Email: ilp@iasbaba.com 0.00 / 160
Page 7
Set 3 Block 2:
Exam Title :
Economics & ...
Email : misrapulkit@yahoo.in
Contact :

Do you know?

• GDP Deflator in India is reported by the The Central Statistics Office (CSO), under
the Ministry of Statistics and Program Implementation.
• GDP of India was 2.264 trillion USD (2016).

THINK!

• GDP of other neighbouring countries and developed countries.


• Measures to increase GDP of India.

Source : NCERT Class 12th- Introductory Macro Economics

QUESTION 7.
Consider the following regarding CPI and GDP deflator

1. CPI includes prices of goods consumed by representative consumer and excludes imported
goods prices whereas GDP deflator includes prices of imported goods

2. In CPI, goods purchased by consumers do not represent all the goods which are produced in
a country whereas GDP deflator takes into account all such goods and services

3. In CPI weights are constant but they differ according to the production level of each good in
GDP deflator.

Select the correct statement/s

a) 1 and 2
b) 2 and 3
c) 1 and 3
d) All of the above
Correct Answer: B
Your Answer: Unanswered
Explanation

Solution (b)

Consumer Price Index is the index of prices of a given basket of commodities which are bought
by the representative consumer. CPI is generally expressed in percentage terms. We have two
years under consideration – one is the base year, the other is the current year. We calculate the
cost of purchase of a given basket of commodities in the base year. We also calculate the cost of
purchase of the same basket in the current year. Then we express the latter as a percentage of
the former. This gives us the Consumer Price Index of the current year vis-à-vis the base year.
For example let us take an economy which produces two goods, rice and cloth. A representative
consumer buys 90 kg of rice and 5 pieces of cloth in a year. Suppose in the year 2000 the price
of a kg of rice was Rs 10 and a piece of cloth was Rs 100. So the consumer had to spend a total
sum of Rs 10 × 90 = Rs 900 on rice in 2000. Similarly, she spent Rs 100 × 5 = Rs 500 per year
on cloth. Summation of the two items is, Rs 900 + Rs 500 = Rs 1,400.

IASbaba
Web: http://ilp.iasbaba.com/ Score:
Email: ilp@iasbaba.com 0.00 / 160
Page 8
Set 3 Block 2:
Exam Title :
Economics & ...
Email : misrapulkit@yahoo.in
Contact :

GDP Deflator(implicit price deflator) is the ratio of nominal to real GDP .Thus if GDP stands
for nominal GDP and gdp stands for real GDP then, GDP deflator =GDP/gdp. Like the consumer
price index (CPI), the GDP deflator is a measure of price inflation/deflation with respect to a
specific base year; the GDP deflator of the base year itself is equal to 100. Unlike the CPI, the
GDP deflator is not based on a fixed basket of goods and services; the "basket" for the GDP
deflator is allowed to change from year to year with people's consumption and investment
patterns. GDP Deflator only takes into account goods that are produced domestically .

Do you know?

• A price index with a fixed basket of goods (CPI) is called a Laspeyres index.
• A price index with a changing basket (GDP deflator)is called Paasche index

THINK!

• Wholesale Price Index


• Producer Price Index
• Cost of living index

Source : NCERT Class 12th- Introductory Macro Economics

IASbaba
Web: http://ilp.iasbaba.com/ Score:
Email: ilp@iasbaba.com 0.00 / 160
Page 9
Set 3 Block 2:
Exam Title :
Economics & ...
Email : misrapulkit@yahoo.in
Contact :

QUESTION 8.
Consider the following about alternative measures of money supply published by RBI

1. M1 and M3 are narrow money.

2. M2 and M4 are broad money.

3. M1 is most liquid and M4 is least liquid of all.

4. M3 is the most commonly used measure of money supply and also known as aggregate
monetary resources.

Choose the correct statement/s

a) 1 ,2 and 3.
b) 1 ,2 and 4.
c) 3 and 4.
d) All of the above.
Correct Answer: C
Your Answer: Unanswered
Explanation

Solution (c)

In economics , the money supply (or money stock) is the total amount of monetary assets availa
ble in an economy at a specific time. There are several ways to define "money", but standard
measures usually include currency in circulation and demand deposits (depositors' easily
accessed assets on the books of financial institutions). Money supply data are recorded and
published, usually by the government or the central bank of the country .

IASbaba
Web: http://ilp.iasbaba.com/ Score:
Email: ilp@iasbaba.com 0.00 / 160
Page 10
Set 3 Block 2:
Exam Title :
Economics & ...
Email : misrapulkit@yahoo.in
Contact :

Money supply, like money demand, is a stock variable. The total stock of money in circulation
among the public at a particular point of time is called money supply. RBI publishes figures for
four alternative measures of money supply viz. M1, M2, M3 and M4. They are defined as follows

M0=Reserve Money

M1 = CU + DD

M2 = M1 + Savings deposits with Post Office savings banks

M3 = M1 + Net time deposits of commercial banks

M4 = M3 + Total deposits with Post Office savings organisations (excluding

National Savings Certificates)

where, CU is currency (notes plus coins) held by the public and DD is net demand deposits held
by commercial banks

M1 and M2 are known as narrow money . M3 and M4 are known as broad money . These
gradations are in decreasing order of liquidity. M1 is most liquid and easiest for transactions

IASbaba
Web: http://ilp.iasbaba.com/ Score:
Email: ilp@iasbaba.com 0.00 / 160
Page 11
Set 3 Block 2:
Exam Title :
Economics & ...
Email : misrapulkit@yahoo.in
Contact :
whereas M4 is least liquid of all. M3 is the most commonly used measure of money supply. It is
also known as aggregate monetary resources.

THINK!

• Supply of Money.
• Monetary aggregates of other countries.

Source : NCERT Class 12th Introductory Macro Economics

QUESTION 9.
Total demand for money in an economy is composed of

a) Transaction demand which is directly proportional to real GDP and speculative demand
which is inversely related to the market rate of interest.
b) Transaction demand which is inversely proportional to real GDP and speculative demand
which is directly related to the market rate of interest Narmada
c) Transaction demand which is directly proportional to market rate of interest and
speculative demand which is inversely related to the real GDP.
d) Transaction demand which is inversely proportional to market rate of interest and
speculative demand which is directly related to the real GDP.
Correct Answer: A
Your Answer: Unanswered
Explanation

Solution (a)

Total demand for money in an economy is composed of transaction demand which is directly
proportional to real GDP and speculative demand which is inversely related to the market
rate of interest. The former is directly proportional to real GDP and price level, whereas the
latter is inversely related to the market rate of interest.

THINK!

• Liquidity Trap.
• Velocity of circulation of money.

Source : NCERT Class 12th - Introductory Macro Economics

QUESTION 10.
Consider the following terms

1. Currency Deposit Ratio- Ratio of money held by public in currency to that they hold in
bank deposits.

2. Reserve Deposit Ratio- Proportion of the total deposits commercial banks keep as
reserves.

3. Statutory Liquidity Ratio- Policy instrument by RBI, which specifies the fraction of their
deposits that bank must keep with RBI.

IASbaba
Web: http://ilp.iasbaba.com/ Score:
Email: ilp@iasbaba.com 0.00 / 160
Page 12
Set 3 Block 2:
Exam Title :
Economics & ...
Email : misrapulkit@yahoo.in
Contact :

4. Cash Reserve Ratio- Policy instrument by RBI, which requires the banks to maintain a
given fraction of their total demand and time deposits in the form of specified liquid assets.

Select the INCORRECT statement/s:

a) 1 and 3
b) 2 and 4
c) 3 and 4
d) None of the above
Correct Answer: C
Your Answer: Unanswered
Explanation

Solution (c)

Money supply will change if the value of any of its components such as CU, DD or time Deposits
changes. Various actions of the monetary authority, RBI, and commercial banks are responsible
for changes in the values of these items. The preference of the public for holding cash balances
vis-à-vis deposits in banks also affect the money supply. These influences on money supply can
be summarised by the following key ratios:

The Currency Deposit Ratio (cdr) : The currency deposit ratio (cdr) is the ratio of money
held by the public in currency to that they hold in bank deposits. cdr = CU/DD. where, CU is
currency (notes plus coins) held by the public and DD is net demand deposits held by
commercial banks.

It reflects people’s preference for liquidity. It is a purely behavioural parameter which depends,
among other things, on the seasonal pattern of expenditure. For example, cdr increases during
the festive season as people convert deposits to cash balance for meeting extra expenditure
during such periods.

The Reserve Deposit Ratio (rdr) : Banks hold a part of the money people keep in their bank
deposits as reserve money and loan out the rest to various investment projects. Reserve money
consists of two things – vault cash in banks and deposits of commercial banks with RBI. Banks
use this reserve to meet the demand for cash by account holders. Reserve deposit ratio (rdr) is
the proportion of the total deposits commercial banks keep as reserves.

Cash Reserve Ratio (crr) : RBI requires commercial banks to keep reserves in order to ensure
that banks have a safe cushion of assets to draw on when account holders want to be paid. RBI
uses various policy instruments to bring forth a healthy rdr in commercial banks. The first
instrument is the Cash Reserve Ratio which specifies the fraction of their deposits that banks
must keep with RBI.

Statutory Liquidity Ratio (slr): RBI requires commercial banks to maintain a given fraction
of their total demand and time deposits in the form of specified liquid assets.

Apart from these ratios RBI uses a certain interest rate called the Bank Rate to control the
value of rdr. Commercial banks can borrow money from RBI at the bank rate when they run
short of reserves. A high bank rate makes such borrowing from RBI costly and, in effect,
encourages the commercial banks to maintain a healthy rdr.

THINK!

• Basel Accords
• Capital adequacy

IASbaba
Web: http://ilp.iasbaba.com/ Score:
Email: ilp@iasbaba.com 0.00 / 160
Page 13
Set 3 Block 2:
Exam Title :
Economics & ...
Email : misrapulkit@yahoo.in
Contact :

Source : NCERT Class 12th - Introductory Macro Economics

QUESTION 11.
Consider the following statements

1. The total liability of the monetary authority of the country is called 'High Powered Money'/
Monetary Base.

2. Currency notes and coins are called 'Fiat money'.

Choose the correct statement/s

a) 1 only
b) 2 only
c) Both
d) None
Correct Answer: C
Your Answer: Unanswered
Explanation

Solution (c)

High Powered Money : The total liability of the monetary authority of the country, RBI, is
called the monetary base or high powered money. It consists of currency (notes and coins in
circulation with the public and vault cash of commercial banks) and deposits held by the
Government of India and commercial banks with RBI. If a member of the public produces a
currency note to RBI the latter must pay her value equal to the figure printed on the note.
Similarly, the deposits are also refundable by RBI on demand from deposit-holders. These items
are claims which the general public, government or banks have on RBI and hence are
considered to be the liability of RBI.

RBI acquires assets against these liabilities. The process can be understood easily if we
consider a simple stylised example. Suppose RBI purchases gold or dollar’s worth Rs 5. It pays
for the gold or foreign exchange by issuing currency to the seller. The currency in circulation in
the economy thus goes up by Rs 5, an item that shows up on the liability side of the balance
sheet. The value of the acquired assets, also equal to Rs 5, is entered under the appropriate
head on the Assets side. Similarly, RBI acquires debt bonds or securities issued by the
government and pays the government by issuing currency in return. It issues loans to
commercial banks in a similar fashion.

FIAT MONEY : Currency notes and coins are called 'Fiat money'. Fiat money is currency that a
government has declared to be legal tender , but it is not backed by a physical commodity. The
value of fiat money is derived from the relationship between supply and demand rather than the
value of the material that the money is made of. Most modern paper currencies are fiat
currencies; they have no intrinsic value and are used solely as a means of payment
Fiat money is inconvertible and cannot be redeemed. Fiat money rose to prominence in the
20th century, specifically after the collapse of the Bretton Woods system in 1971, when the
United States ceased to allow the conversion of the dollar into gold

IASbaba
Web: http://ilp.iasbaba.com/ Score:
Email: ilp@iasbaba.com 0.00 / 160
Page 14
Set 3 Block 2:
Exam Title :
Economics & ...
Email : misrapulkit@yahoo.in
Contact :

THINK!

• Cryptocurrency
• Seigniorage
• Fractional-reserve banking

Source : NCERT Class 12th- Introductory Macro Economics

QUESTION 12.
Which one is/are considered as a 'Lender of last resort'?

a) Scheduled Banks
b) Commercial Banks
c) Government of India
d) Reserve Bank of India
Correct Answer: D
Your Answer: Unanswered
Explanation

Solution (d)

RBI is the correct answer.

Scheduled Banks : refers to a bank which is listed in the 2nd Schedule of the Reserve Bank of
India Act, 1934. Banks not under this Schedule are called non-scheduled banks. Scheduled
banks are usually private, foreign and nationalised banks operating in India.

IASbaba
Web: http://ilp.iasbaba.com/ Score:
Email: ilp@iasbaba.com 0.00 / 160
Page 15
Set 3 Block 2:
Exam Title :
Economics & ...
Email : misrapulkit@yahoo.in
Contact :

IASbaba
Web: http://ilp.iasbaba.com/ Score:
Email: ilp@iasbaba.com 0.00 / 160
Page 16
Set 3 Block 2:
Exam Title :
Economics & ...
Email : misrapulkit@yahoo.in
Contact :

Commercial banks are banks which are directly dealing with customers - opening deposit
accounts, lending loans, issuing demand drafts, sending online remittances etc., and the
customers can approach these bank people and get their services fulfilled.

Unlike commercial banks, development banks like NABARD do not have deposit or loan
accounts and they will simply process the loan proposals received from the customers, sanction
the loans and the amount will be disbursed through a commercial bank like SBI.

The Reserve Bank of India (RBI) is India's central banking institution, which controls the mo
netary policy of the Indian rupee . It commenced its operations on 1 April 1935 during the
British Rule in accordance with the provisions of the Reserve Bank of India Act, 1934 . A
Central Bank is an independent apex monetary authority which regulates banks and provides
important financial services like storing of foreign exchange reserves, control of inflation,
monetary policy report. A Central Bank is known by different names in different countries.

THINK!

• Co-operative banks
• NABARD, HUDCO etc

Source : NCERT Class 12th - Introductory Macro Economics

QUESTION 13.
Consider the following regarding Revenue and Expenditure measures of the government

1. Allocation Function- Public goods must be provided by government.

2. Distribution Function- Government affects the personal disposable income of households by


making transfer payments and collecting taxes.

3. Public Provision- These are financed through budget and made available free of any direct
payment.

Choose the correct options

a) 1 only
b) 1 and 2 only
c) 2 and 3 only
d) All of the above
Correct Answer: D
Your Answer: Unanswered
Explanation

Solution (d)

Certain goods, referred to as public goods (such as national defence, roads, government
administration), as distinct from private goods (like clothes, cars, food items), cannot be
provided through the market mechanism, i.e. by transactions between individual consumers
and producers and must be provided by the government. This is the allocation function .

Through its tax and expenditure policy, the government attempts to bring about a distribution
of income that is considered ‘fair’ by society. The government affects the personal disposable

IASbaba
Web: http://ilp.iasbaba.com/ Score:
Email: ilp@iasbaba.com 0.00 / 160
Page 17
Set 3 Block 2:
Exam Title :
Economics & ...
Email : misrapulkit@yahoo.in
Contact :
income of households by making transfer payments and collecting taxes and, therefore, can
alter the income distribution. This is the distribution function .

Public provision means that they are financed through the budget and made available free of
any direct payment. These goods may be produced directly under government management or
by the private sector.

THINK!

• Free- Rider Problem.


• Mixed Economy

Source : NCERT Class 12th - Introductory Macro Economics

QUESTION 14.
Consider the following statements

1. The total amount of money stock in the economy is much greater than the volume of 'High
powered money'.

2. The ratio of stock of high powered money to stock of money in an economy is called 'Money
multiplier'.

Choose the correct statement/s

a) 1 only
b) 2 only
c) Both
d) None
Correct Answer: A
Your Answer: Unanswered
Explanation

Solution (a)

High Powered Money : The total liability of the monetary authority of the country, RBI, is
called the monetary base or high powered money. It consists of currency (notes and coins in
circulation with the public and vault cash of commercial banks) and deposits held by the
Government of India and commercial banks with RBI. If a member of the public produces a
currency note to RBI the latter must pay her value equal to the figure printed on the note.
Similarly, the deposits are also refundable by RBI on demand from deposit-holders. These items
are claims which the general public, government or banks have on RBI and hence are
considered to be the liability of RBI

The total amount of money stock in the economy is much greater than the volume of high
powered money. Commercial banks create this extra amount of money by giving out a part of
their deposits as loans or investment credits. The total amount of deposits held by all
commercial banks in the country is much larger than the total size of their reserves.

Money Multiplier: The ratio of the stock of money to the stock of high powered money in an
economy. Its value is greater than 1. It is one of various closely related ratios of commercial
bank money to central bank money under a fractional-reserve banking system. It measures an

IASbaba
Web: http://ilp.iasbaba.com/ Score:
Email: ilp@iasbaba.com 0.00 / 160
Page 18
Set 3 Block 2:
Exam Title :
Economics & ...
Email : misrapulkit@yahoo.in
Contact :
estimate of the maximum amount of commercial bank money that can be created, given a
certain amount of central bank money. That is, in a fractional-reserve banking system, the total
amount of loans that commercial banks are allowed to extend (the commercial bank money that
they can legally create) is equal to an amount which is a multiple of the amount of reserves.

THINK!

• Fractional-reserve banking system.


• Economic multiplier.

Source : NCERT Class 12th- Introductory Macro Economics

QUESTION 15.
Consider the following statements

1. Ex Ante measures are the actual values of consumption and investment variables.

2. Ex Post measures are the planned values of consumption and investment variables.

Choose the correct statement/s

a) 1 only
b) 2 only
c) Both
d) None
Correct Answer: D
Your Answer: Unanswered
Explanation

Solution (d)

Ex-Ante: Ex-ante, derived from the Latin for "before the event," is a term that refers to future
events, such as future returns or prospects of a company. Ex-ante analysis helps to give an idea
of future movements in price or the future impact of a newly implemented policy. An example of
ex-ante analysis is when an investment company values a stock ex ante and then compares the
predicted results to the actual movement of the stock's price.

Ex Ante measures are the planned values of consumption and investment variables.

"Ex-ante" refers to any prediction that is made prior to either before all of the variables are
known, or generally before an event occurs. For example, an ex-ante price on a stock is an
informed estimate functioning as a prediction of future events when not all variables are
known. It provides for a predictive model, allowing for the uncertainty surrounding current
conditions or any unknown factor that may change the outcome.

Ex-Post: Ex-post is another word for actual returns and is Latin for "after the fact." The use of h
istorical returns has customarily been the most well-known approach to forecast the probability
of incurring a loss on an investment on any given day. Ex-post is the opposite of ex-ante , which
means "before the event."

Ex Post measures are the actual values of consumption and investment variables. Companies
attain ex-post data to forecast future earnings . Ex-post data is utilized in studies such as value
at risk (VaR), a probability study that approximates the maximum amount of loss an investment

IASbaba
Web: http://ilp.iasbaba.com/ Score:
Email: ilp@iasbaba.com 0.00 / 160
Page 19
Set 3 Block 2:
Exam Title :
Economics & ...
Email : misrapulkit@yahoo.in
Contact :
portfolio may incur on any day. VaR is defined for a specified investment portfolio, probability
and time horizon

Ex-post yield differs from ex-ante yield because it represents actual values, essentially what
investors earn rather than estimated values. Investors base their decisions on expected returns
versus actual returns, which is an important aspect of an investment's risk analysis . Ex-post is
the current market price , minus the price the investor paid. It shows the performance of an
asset; however, it excludes projections and probabilities

THINK!

• Ex-ante consumption and investment


• Effective demand principle

Source : NCERT Class 12th - Introductory Macro Economics

QUESTION 16.
Consider the following terms related to Government Budget

1. Capital Receipts- Receipts of the government which are Non-redeemable.

2. Capital Expenditure- Expenses incurred for the purposes other than the creation of
physical or financial assets.

3. Revenue Expenditure- Expenses of the government which result in physical or financial


assets.

4. Revenue Receipts- Receipts of the government which create liability.

Choose the incorrect options

a) 1,2, 3 and 4
b) 3 and 4 only
c) 1 and 4 only
d) 1 and 2 only
Correct Answer: A
Your Answer: Unanswered
Explanation

Solution (a)

There is a constitutional requirement in India (Article 112) to present before the Parliament a
statement of estimated receipts and expenditures of the government in respect of every
financial year which runs from 1 April to 31 March. This ‘Annual Financial Statement’
constitutes the main budget document. The Government presents it on the first day of February
so that it could be materialized before the commencement of new financial year in April. Till
2016 it was presented on the last working day of February by the Finance Minister of
India in Parliament. The budget, which is presented by means of the Finance bill and the
Appropriation bill has to be passed by both the Houses before it can come into effect from April
1, the start of India's financial year. Further, the budget must distinguish expenditure on the
revenue account from other expenditures. Therefore, the budget comprises of the (a) Revenue
Budget and the (b) Capital Budget.

IASbaba
Web: http://ilp.iasbaba.com/ Score:
Email: ilp@iasbaba.com 0.00 / 160
Page 20
Set 3 Block 2:
Exam Title :
Economics & ...
Email : misrapulkit@yahoo.in
Contact :

The Revenue Budget shows the current receipts of the government and the expenditure that
can be met from these receipts

Revenue Receipts : Revenue receipts are receipts of the government which are non-
redeemable, that is, they cannot be reclaimed from the government. They are divided into tax
and non-tax revenues. Non-tax revenue of the central government mainly consists of interest
receipts on account of loans by the central government, dividends and profits on investments
made by the government, fees and other receipts for services rendered by the government.
Cash grants-in-aid from foreign countries and international organisations are also included.

The estimates of revenue receipts take into account the effects of tax proposals made in the
Finance Bill.

Revenue Expenditure : Revenue Expenditure is expenditure incurred for purposes other than
the creation of physical or financial assets of the central government. It relates to those
expenses incurred for the normal functioning of the government departments and various
services, interest payments on debt incurred by the government, and grants given to state
governments and other parties (even though some of the grants may be meant for creation of
assets).

The Capital Budget is an account of the assets as well as liabilities of the central government,
which takes into consideration changes in capital. It consists of capital receipts and capital
expenditure of the government. This shows the capital requirements of the government and the
pattern of their financing.

Capital Receipts : All those receipts of the government which create liability or reduce
financial assets are termed as capital receipts. The main items of capital receipts are loans
raised by the government from the public which are called market borrowings, borrowing by
the government from the Reserve Bank and commercial banks and other financial institutions
through the sale of treasury bills, loans received from foreign governments and international
organisations, and recoveries of loans granted by the central government. Other items include
small savings (Post-Office Savings Accounts, National Savings Certificates, etc), provident funds
and net receipts obtained from the sale of shares in Public Sector Undertakings (PSUs) (This is
referred to as PSU disinvestment).

Capital Expenditure : There are expenditures of the government which result in creation of
physical or financial assets or reduction in financial liabilities. This includes expenditure on the
acquisition of land, building, machinery, equipment, investment in shares, and loans and
advances by the central government to state and union territory governments, PSUs and other
parties.

THINK!

• Government Deficits
• Halwa Ceremony

Source : NCERT Class 12th - Introductory Macro Economics

QUESTION 17.
The term 'Fiscal Deficit' is defined as

a) Difference between governments total receipts and total expenditure excluding borrowing.

IASbaba
Web: http://ilp.iasbaba.com/ Score:
Email: ilp@iasbaba.com 0.00 / 160
Page 21
Set 3 Block 2:
Exam Title :
Economics & ...
Email : misrapulkit@yahoo.in
Contact :

b) Difference between governments total receipts and total expenditure including borrowing.
c) Difference between governments total expenditure and total receipts excluding borrowing.
d) Difference between governments total expenditure and total receipts including borrowing.
Correct Answer: C
Your Answer: Unanswered
Explanation

Solution (c)

Fiscal Deficit : Fiscal deficit is the difference between the government’s total expenditure and
its total receipts excluding borrowing .Gross fiscal deficit = Total expenditure – (Revenue
receipts + Non-debt creating capital receipts).

Deficit differs from debt, which is an accumulation of yearly deficits. A fiscal deficit is regarded
by some as a positive economic event. For example, economist John Maynard Keynes believed
that deficits help countries climb out of economic recession . On the other hand, fiscal
conservatives feel that governments should avoid deficits in favour of a balanced budget policy.

The fiscal deficit will have to be financed through borrowing. Thus, it indicates the total
borrowing requirements of the government from all sources. From the financing side

Gross fiscal deficit = Net borrowing at home + Borrowing from RBI +Borrowing from abroad
Net borrowing at home includes that directly borrowed from the public through debt
instruments (for example, the various small savings schemes) and indirectly from commercial
banks through Statutory Liquidity Ratio (SLR). The gross fiscal deficit is a key variable in
judging the financial health of the public sector and the stability of the economy.

Do you know?

• For this fiscal 2017-18, the government has set a target of 3.2% of the GDP .
• India’s fiscal deficit in July end came at Rs 5.05 lakh crore or 92.4% of the budgetary
estimates for 2017-18, as against 73.7% in the corresponding period in the last fiscal .

THINK!

• Revenue Deficit
• Primary Deficit

Source : NCERT Class 12th - Introductory Macro Economics

QUESTION 18.
What is true about 'Ricardian Equivalence'?

a) During high deficits, people spend more and consider taxation and borrowing as equivalent
means of financing the expenditure.
b) During high deficits, people save more and considers taxation and borrowing as equivalent
means of financing the expenditure.
c) During high deficits, people spend more and consider taxation and investment as equivalent
means of financing the expenditure.
d) During high deficits, people save more and considers taxation and investment as equivalent
means of financing the expenditure.

IASbaba
Web: http://ilp.iasbaba.com/ Score:
Email: ilp@iasbaba.com 0.00 / 160
Page 22
Set 3 Block 2:
Exam Title :
Economics & ...
Email : misrapulkit@yahoo.in
Contact :
Correct Answer: B
Your Answer: Unanswered
Explanation

Solution (b)

Ricardian Equivalence :

David Ricardo, who first argued that in the face of high deficits, people save more. It is called
‘equivalence’ because it argues that taxation and borrowing are equivalent means of financing
expenditure. When the government increases spending by borrowing today, which will be
repaid by taxes in the future, it will have the same impact on the economy as an increase in
government expenditure that is financed by a tax increase today. Governments can finance their
expenditures either through taxes or by issuing bonds. Since bonds are loans, they must
eventually be repaid—presumably by raising taxes in the future. The choice is therefore "tax
now or tax later."

Suppose that the government finances some extra spending through deficits; i.e. it chooses to
tax later. According to the hypothesis, taxpayers will anticipate that they will have to pay higher
taxes in future. As a result, they will increase their savings to pay the future tax increase; i.e.
they reduce their current consumption to do so. The effect on aggregate demand would be the
same as if the government had chosen to tax now.

THINK!

• Say's law
• Fiscal Policy

Source : NCERT Class 12th - Introductory Macro Economics

QUESTION 19.
Consider the following regarding government debt

1. Debt is burdensome if it reduces future growth in output.

2. Debt is not a burden when it is owed to foreigners since we have to lend goods abroad
corresponding to interest payments.

Choose the correct statement/s

a) 1 only
b) 2 only
c) Both
d) None
Correct Answer: A
Your Answer: Unanswered
Explanation

Solution (a)

There are two interlinked aspects of the Perspectives on the appropriate amount of Government
Debt. One is whether government debt is a burden and two is regarding the issue of financing
the debt.

IASbaba
Web: http://ilp.iasbaba.com/ Score:
Email: ilp@iasbaba.com 0.00 / 160
Page 23
Set 3 Block 2:
Exam Title :
Economics & ...
Email : misrapulkit@yahoo.in
Contact :

It has often been argued that public ‘debt does not matter because we owe it to ourselves’. This
is because although there is a transfer of resources between generations, purchasing power
remains within the nation. However, any debt that is owed to foreigners involves a burden since
we have to send goods abroad corresponding to the interest payments.

If the government invests in infrastructure, future generations may be better off, provided the
return on such investments is greater than the rate of interest. The actual debt could be paid off
by the growth in output. The debt should not then be considered burdensome. The growth in
debt will have to be judged by the growth of the economy as a whole. Hence Debt is
burdensome if it reduces future growth in output.

THINK!

• Govt Deficits and Debt.

Source : NCERT Class 12th - Introductory Macro Economics

QUESTION 20.
Choose the correct statement regarding FRBMA- 2003

a) It is an act of a parliament of India which mandates Finance Minister to prepare Annual


Financial Statement every year.
b) It is an act of Government of India to institutionalize financial discipline, to increase India's
Fiscal Deficit and to eliminate Revenue Deficit
c) It is an act of RBI which commits to bring down Fiscal Deficit and eliminate Revenue
deficits.
d) It is an act of Parliament of India to institutionalize financial discipline, to reduce India's
Fiscal Deficit and to eliminate Revenue Deficit.
Correct Answer: D
Your Answer: Unanswered
Explanation

Solution (d)

The Fiscal Responsibility and Budget Management Act, 2003 (FRBMA) is an Act of
the Parliament of India to institutionalize financial discipline, reduce India's fiscal deficit,
improve macroeconomic management and the overall management of the public funds by
moving towards a balanced budget and strengthen fiscal prudence. The main purpose was to
eliminate revenue deficit of the country (building revenue surplus thereafter) and bring down
the fiscal deficit to a manageable 3% of the GDP by March 2008.

The Fiscal Responsibility and Budget Management Bill (FRBM Bill) was introduced in India by
the then Finance Minister of India, Mr.Yashwant Sinha in December 2000. Firstly, the bill
highlighted the terrible state of government finances in India both at the Union and the state
levels under the statement of objects and reasons. Secondly, it sought to introduce the
fundamentals of fiscal discipline at the various levels of the government

The main objectives of the act were

• to introduce transparent fiscal management systems in the country.

IASbaba
Web: http://ilp.iasbaba.com/ Score:
Email: ilp@iasbaba.com 0.00 / 160
Page 24
Set 3 Block 2:
Exam Title :
Economics & ...
Email : misrapulkit@yahoo.in
Contact :
• to introduce a more equitable and manageable distribution of the country's debts over the
years .
• to aim for fiscal stability for India in the long run.

Additionally, the act was expected to give necessary flexibility to Reserve Bank of India (RBI) for
managing inflation in India

Economic Advisory Council publicly advised the Government of India to reconsider reinstating
the provisions of the FRBMA. N. K. Singh is currently the Chairman of the review committee for
Fiscal Responsibility and Budget Management Act, 2003, under the Ministry of Finance
(India), Government of India.

Documents presented under FRBM

• Medium Term Fiscal Policy :sets out three-year rolling targets for five specific fiscal
indicators in relation to GDP at market prices, namely, (i) Revenue Deficit (ii) effective
revenue deficit, (iii) Fiscal Deficit, (iv) Tax to GDP ratio and (v) Total outstanding Debt as
percentage of GDP at the end of the year.
• Fiscal Policy Strategy Statement: the strategic priorities of Government in the fiscal area
for the ensuing financial year relating to taxation, expenditure, lending and investments,
administered pricing, borrowings and guarantees in moving towards the FRBM targets.
• Macroeconomic Framework Statement :an assessment regarding the expected GDP
growth rate, fiscal balance of the Central Government and the external sector balance of
the economy

THINK!

• FRBMA-2012
• FRBM Review Committee of 2016

Source : NCERT Class 12th - Introductory Macro Economics

QUESTION 21.
Consider the following statements

1. An Open Economy is the one that trades with other nations only with its services.

2. GDP is a common measure of degree of openness of an economy.

3. GNP is a common measure of degree of openness of an economy.

Choose the correct statement/s

a) 1 only
b) 2 and 3 only
c) 2 only
d) 1 and 2 only
Correct Answer: C
Your Answer: Unanswered
Explanation

Solution (c)

IASbaba
Web: http://ilp.iasbaba.com/ Score:
Email: ilp@iasbaba.com 0.00 / 160
Page 25
Set 3 Block 2:
Exam Title :
Economics & ...
Email : misrapulkit@yahoo.in
Contact :

An open economy is one that trades with other nations in goods and services and, most often,
also in financial assets. It is an economy in which there are economic activities between the
domestic community and outside. People and even businesses can trade in goods and services w
ith other people and businesses in the international community , and funds can flow as
investments across the border. Trade can take the form of managerial exchange, technology
transfers , and all kinds of goods and services. (However, certain exceptions exist that cannot
be exchanged; the railway services of a country, for example, cannot be traded with another
country to avail the service).Indians, for instance, enjoy using products produced around the
world and some of our production is exported to foreign countries. Foreign trade, therefore,
influences Indian aggregate demand in two ways:

• First, when Indians buy foreign goods, this spending escapes as a leakage from the
circular flow of income decreasing aggregate demand.
• Second, our exports to foreigners enter as an injection into the circular flow, increasing
aggregate demand for domestically produced goods.

Total foreign trade (exports + imports) as a proportion of GDP is a common measure of the
degree of openness of an economy. In 2013-14, this was 44.1 per cent for the Indian
Economy. There are several countries whose foreign trade proportions are above 50 per cent of
GDP.

IASbaba
Web: http://ilp.iasbaba.com/ Score:
Email: ilp@iasbaba.com 0.00 / 160
Page 26
Set 3 Block 2:
Exam Title :
Economics & ...
Email : misrapulkit@yahoo.in
Contact :

Now, when goods move across national borders, money must move in the opposite direction. At
the international level, there is no single currency that is issued by a central authority. Foreign
economic agents will accept a national currency only if they are convinced that the currency
will maintain a stable purchasing power. Without this confidence, a currency will not be used as
an international medium of exchange and unit of account since there is no international
authority with the power to force the use of a particular currency in international transactions.

Governments have tried to gain confidence of potential users by announcing that the national
currency will be freely convertible at a fixed price into another asset, over whose value the
issuing authority has no control. This other asset most often has been gold, or other national
currencies. There are two aspects of this commitment that has affected its credibility – the
ability to convert freely in unlimited amounts and the price at which conversion takes place.
The international monetary system has been set up to handle these issues and ensure
stability in international transactions. A nation’s commitment regarding the above two issues
will affect its trade and financial interactions with the rest of the world

THINK!

• Closed economy

Source : NCERT Class 12th - Introductory Macro Economics

QUESTION 22.
Choose the correct statements regarding Balance Of Payments

a) It records all the transactions about goods, services and assets between residents of a
country with the rest of the world for a specified period.
b) It records all the transactions about goods, services and assets between Citizens of India
with the rest of the world for a specified period.
c) It records all the transactions about goods, services and assets between Citizens of India
and Indian residents in other countries for a specified period.
d) It records all the transactions about goods, services and assets between Citizens of India
and foreigners residing in India for a specified period.
Correct Answer: A
Your Answer: Unanswered
Explanation

Solution (a)

The BALANCE OF PAYMENTS (BOP) records the transactions in goods, services and assets
between residents of a country with the rest of the world for a specified time period typically a
year. These transactions are made by individuals, firms and government bodies. Thus the
balance of payments includes all external visible and non-visible transactions of a country. It is
an important issue to be studied, especially in international financial management field, for a
few reasons:

• First, the balance of payments provides detailed information concerning the demand and
supply of a country's currency. For example, if India imports more than it exports, then
this means that the supply of rupees is likely to exceed the demand in the foreign
exchanging market. One can thus infer that the Indian rupee would be under pressure to

IASbaba
Web: http://ilp.iasbaba.com/ Score:
Email: ilp@iasbaba.com 0.00 / 160
Page 27
Set 3 Block 2:
Exam Title :
Economics & ...
Email : misrapulkit@yahoo.in
Contact :
depreciate against other currencies. On the other hand, if India's exports more than it
imports, then the rupee would be likely to appreciate.
• Second, a country's balance-of-payment data may signal its potential as a business partner
for the rest of the world. If a country is grappling with a major balance-of-payment
difficulty, it may not be able to expand imports from the outside world.

There are two main accounts in the BoP – the current account and the capital account

The current account records exports and imports in goods and services and transfer
payments. When exports exceed imports, there is a trade surplus and when imports exceed
exports there is a trade deficit. Trade in services denoted as invisible trade (because they are
not seen to cross national borders) includes both factor income (net income from compensation
of employees and net investment income, the latter equals, the interest, profits and dividends
on our assets abroad minus the income foreigners earn on assets they own in India) and net
non-factor income (shipping, banking, insurance, tourism, software services, etc.). Transfer
payments are receipts which the residents of a country receive ‘for free’, without having to
make any present or future payments in return. They consist of remittances, gifts and grants.
They could be official or private. The balance of exports and imports of goods is referred to as
the trade balance.

The capital account records all international purchases and sales of assets such as money,
stocks, bonds, etc. We note that any transaction resulting in a payment to foreigners is entered
as a debit and is given a negative sign. Any transaction resulting in a receipt from foreigners is
entered as a credit and is given a positive sign.

IASbaba
Web: http://ilp.iasbaba.com/ Score:
Email: ilp@iasbaba.com 0.00 / 160
Page 28
Set 3 Block 2:
Exam Title :
Economics & ...
Email : misrapulkit@yahoo.in
Contact :

THINK!

• BOP Surplus
• BOP Deficit

Source : NCERT Class 12th - Introductory Macro Economics

QUESTION 23.
Consider the following regarding autonomous and accommodating transactions

1. Autonomous transactions are called 'Above the line' items in Balance of Payments whereas
Accommodating transactions are called 'Below the Line' items in Balance of Payments.

2. Autonomous transactions are International economic transactions whereas Accommodating


transactions are official reserve transactions.

Choose the correct statement/s

a) 1 only
b) 2 only
c) Both
d) None
Correct Answer: C
Your Answer: Unanswered
Explanation

Solution (c)

The basic difference between the Autonomous Transactions and Accommodating Transactions
is that deficit or surplus in Balance Of Payments-BOP occurs due to autonomous items, the
accommodating items are taken to cover deficit (or surplus) in autonomous transactions

IASbaba
Web: http://ilp.iasbaba.com/ Score:
Email: ilp@iasbaba.com 0.00 / 160
Page 29
Set 3 Block 2:
Exam Title :
Economics & ...
Email : misrapulkit@yahoo.in
Contact :

Autonomous Transactions :

These refer to international economic transactions that take place due to some economic
motives like earning income and profit maximisation. They have nothing to do with foreign
exchange payments. Since such transactions are independent of the state of country’s balance
of payment, i.e., irrespective of whether BOP is favourable or unfavourable, they are, therefore,
called autonomous items. These items are called ‘above the line’ items in the BoP. The balance
of payments is said to be in surplus (deficit) if autonomous receipts are greater (less) than
autonomous payments.

Accomodating Transactions :

Accommodating transactions (termed ‘below the line’ items), on the other hand, are determined
by the net consequences of the autonomous items, that is, whether the BoP is in surplus or
deficit.

These refer to transactions that take place to cover deficit (or surplus) arising from autonomous
transactions. These items are also called ‘below the line items’. Because of government
financing, official settlements are seen as accommodating items to keep the BOP identity. The
official reserve transactions are seen as the accommodating item in the BoP (all others being
autonomous).

THINK!

• Other parameters in BOP

Source : NCERT Class 12th - Introductory Macro Economics

IASbaba
Web: http://ilp.iasbaba.com/ Score:
Email: ilp@iasbaba.com 0.00 / 160
Page 30
Set 3 Block 2:
Exam Title :
Economics & ...
Email : misrapulkit@yahoo.in
Contact :

QUESTION 24.
Consider the following terms related to Exchange rates

1. Flexible Exchange rate- It is determined by the forces of market demand and supply.

2. Nominal Exchange rate-Exchange rates are quoted in money terms.

3. Real Exchange rate- Ratio of domestic to foreign prices measured in same currency.

Choose the correct options

a) 1 and 2
b) 2 and 3
c) 1 and 3
d) All of the above
Correct Answer: A
Your Answer: Unanswered
Explanation

Solution (a)

Flexible Exchange Rates: Also known as Floating Exchange rate which is determined by the
forces of market demand and supply. In a completely flexible system, the central banks follow a
simple set of rules – they do nothing to directly affect the level of the exchange rate, in other
words they do not intervene in the foreign exchange market (and therefore, there are no official
reserve transactions).

Nominal Exchange Rate: an exchange rate is the rate at which one currency will be
exchanged for another. It is also regarded as the value of one country’s currency in relation to
another currency. It is also called as Bilateral nominal exchange rate, bilateral in the sense that
they are exchange rates for one currency against another and they are nominal because they
quote the exchange rate in money terms, i.e. so many rupees per dollar or per pound.

Real Exchange Rate: The ratio of foreign to domestic prices, measured in the same currency.
It is the purchasing power of a currency relative to another at current exchange rates and
prices. It is the ratio of the number of units of a given country's currency necessary to buy a
market basket of goods in the other country, after acquiring the other country's currency in the
foreign exchange market, to the number of units of the given country's currency that would be
necessary to buy that market basket directly in the given country.

THINK!

• Foreign exchange market


• Effective exchange rate

Source : NCERT Class 12th - Introductory Macro Economics

QUESTION 25.
Consider the following statements regarding Purchasing Power Parity (PPP)

IASbaba
Web: http://ilp.iasbaba.com/ Score:
Email: ilp@iasbaba.com 0.00 / 160
Page 31
Set 3 Block 2:
Exam Title :
Economics & ...
Email : misrapulkit@yahoo.in
Contact :

1. It is used to make long run predictions about real exchange rate system.

2. It states that the exchange rate between one currency and another currency is in equilibrium
when their domestic purchasing powers at that rate of exchange are equivalent

3. If one country has higher inflation than another country , formers exchange rate should
depreciate

4. It was propounded by professor Gustav Cassel of Sweden.

Choose the correct statement/s

a) 1, 2 and 3
b) 2, 3 and 4
c) 1,3 and 4
d) 1,2 and 4
Correct Answer: B
Your Answer: Unanswered
Explanation

Solution (b)

Purchasing power parity (PPP) is an economic theory which was propounded by professor
Gustav Cassel of Sweden that states that the exchange rate between two currencies is equal to
the ratio of the currencies respective purchasing power . The concept of purchasing power
parity allows one to estimate what the exchange rate between two currencies would have to be
in order for the exchange to be at par with the purchasing power of the two countries'
currencies.

If the real exchange rate is equal to one, currencies are at purchasing power parity. This means
that goods cost the same in two countries when measured in the same currency. For instance, if
a pen costs $4 in the US and the nominal exchange rate is Rs 50 per US dollar, then with a real
exchange rate of 1, it should cost Rs 200 (ePf = 50 × 4) in India. If the real exchange rises
above 1, this means that goods abroad have become more expensive than goods at home.

It states that the exchange rate between one currency and another currency is in equilibrium
when their domestic purchasing powers at that rate of exchange are equivalent. If one country
has higher inflation than another country , formers exchange rate should depreciate.

IASbaba
Web: http://ilp.iasbaba.com/ Score:
Email: ilp@iasbaba.com 0.00 / 160
Page 32
Set 3 Block 2:
Exam Title :
Economics & ...
Email : misrapulkit@yahoo.in
Contact :

PPP exchange rates help costing but exclude profits. So, it is reckoned as more efficient
methodology than the use of market exchange rates. For example, suppose that two countries
produce the same physical amounts of goods as each other in each of two different years. Since
market exchange rates fluctuate substantially, when the GDP of one country measured in its
own currency is converted to the other country's currency using market exchange rates, one
country might be inferred to have higher real GDP than the other country in one year but lower
in the other; both of these inferences would fail to reflect the reality of their relative levels of
production. But if one country's GDP is converted into the other country's currency using PPP
exchange rates instead of observed market exchange rates, the false inference will not occur.

THINK!

• KFC Index
• BIG MAC Index
• IPAD Index

Source : NCERT Class 12th - Introductory Macro Economics

QUESTION 26.

IASbaba
Web: http://ilp.iasbaba.com/ Score:
Email: ilp@iasbaba.com 0.00 / 160
Page 33
Set 3 Block 2:
Exam Title :
Economics & ...
Email : misrapulkit@yahoo.in
Contact :

System in which exchange rate is maintained by Monetary authorities of a country is termed as

a) Nominal effective exchange rate.


b) Real effective exchange rate.
c) Pegged exchange rate.
d) Flexible exchange rate.
Correct Answer: C
Your Answer: Unanswered
Explanation

Solution (c)

Nominal Effective Exchange Rate (NEER) which is a multilateral rate representing the price
of a representative basket of foreign currencies, each weighted by its importance to the
domestic country in international trade (the average of export and import shares is taken as an
indicator of this).

Real Effective Exchange Rate (REER) is calculated as the weighted average of the real
exchange rates of all its trade partners, the weights being the shares of the respective countries
in its foreign trade. It is interpreted as the quantity of domestic goods required to purchase one
unit of a given basket of foreign goods.

Flexible exchange rates (FER) (also known as floating exchange rates), the exchange rate is
determined by the forces of market demand and supply. In a completely flexible system, the
central banks follow a simple set of rules – they do nothing to directly affect the level of the
exchange rate, in other words they do not intervene in the foreign exchange market (and
therefore, there are no official reserve transactions).

Pegged exchange rate system (PERS) is a system in which exchange rate is pegged at a
particular level. Sometimes, a distinction is made between the fixed and pegged exchange
rates. It is argued that while the former is fixed, the latter is maintained by the monetary
authorities, in that the value at which the exchange rate is pegged (the par value) is a policy
variable – it may be changed.

THINK!

• Effective exchange rate


• Devaluation
• Revaluation

Source : NCERT Class 12th - Introductory Macro Economics

QUESTION 27.
Consider the following regarding 'Dirty Floating'

1. It is also called as Managed floating exchange rate system.

2. It is a mixture of flexible exchange rate system and a fixed rate system.

Choose the correct statement/s

a) 1 only

IASbaba
Web: http://ilp.iasbaba.com/ Score:
Email: ilp@iasbaba.com 0.00 / 160
Page 34
Set 3 Block 2:
Exam Title :
Economics & ...
Email : misrapulkit@yahoo.in
Contact :

b) 2 only
c) Both
d) None
Correct Answer: C
Your Answer: Unanswered
Explanation

Solution (c)

Managed Floating: Without any formal international agreement, the world has moved on to
what can be best described as a managed floating exchange rate system. It is the current intern
ational financial environment in which exchange rates fluctuate from day to day, but central
banks attempt to influence their countries' exchange rates by buying and selling currencies . It
is also known as a Dirty Floating . It is a mixture of a flexible exchange rate system (the float
part) and a fixed rate system (the managed part).

Under this system central banks intervene to buy and sell foreign currencies in an attempt to
moderate exchange rate movements whenever they feel that such actions are appropriate.
Official reserve transactions are, therefore, not equal to zero. In an increasingly integrated
world economy, the currency rates impact any given country's economy through the trade
balance . In this aspect, almost all currencies are managed since central banks or governments
intervene to influence the value of their currencies.

THINK!

• Fixed exchange rate


• December Mistake
• Black Wednesday

Source : NCERT Class 12th - Introductory Macro Economics

QUESTION 28.
Consider the following statements regarding Bretton Woods Conference

1. It was held in 1944 formally known as United nations monetary and financial order.

2. It created IBRD in order to promote stability of exchange rates and financial flows.

3. It created IMF to speedup reconstruction after second world war and to lend for building
infrastructure.

4. It recommended for International Economic cooperation.

Choose the correct statement/s

a) 1 and 2
b) 2 and 3
c) 1 and 4
d) All of the above
Correct Answer: C
Your Answer: Unanswered
Explanation

IASbaba
Web: http://ilp.iasbaba.com/ Score:
Email: ilp@iasbaba.com 0.00 / 160
Page 35
Set 3 Block 2:
Exam Title :
Economics & ...
Email : misrapulkit@yahoo.in
Contact :

Solution (c)

The Bretton Woods System : The Bretton Woods Conference formally known as the ' United N
ations Monetary and Financial Conference' , was the gathering of 730 delegates from all 44 Alli
ed nations at the Mount Washington Hotel , situated in Bretton Woods , New Hampshire ,
United States, to regulate the international monetary and financial order after the conclusion of
World War II . It was held in 1944 set up the International Monetary Fund (IMF) and the World
Bank and re-established a system of fixed exchange rates. This was different from the
international gold standard in the choice of the asset in which national currencies would be
convertible.

The Bretton Woods Conference had three main results:

(1) Articles of Agreement to create the IMF, whose purpose was to promote stability of
exchange rates and financial flows.

(2) Articles of Agreement to create the IBRD, whose purpose was to speed reconstruction after
the Second World War and to foster economic development, especially through lending to build
infrastructure.

(3) Other recommendations for international economic cooperation. The Final Act of the
conference incorporated these agreements and recommendations.

The seminal idea behind the Bretton Woods Conference was the notion of open markets .

THINK!

• International Monetary Systems


• International Trade Organisations
• Calls for a 'New Bretton Woods'

Source : NCERT Class 12th - Introductory Macro Economics

QUESTION 29.
Consider the following statements regarding Real Exchange Rate system

1. It is the relative price of domestic goods in terms of foreign goods.

2. It measures the International competitiveness of a country in International trade.

3. Any two countries are said to be in Purchasing Power Parity when their real exchange rate is
equal to one.

Choose the correct statement/s

a) 1 and 2
b) 2 and 3
c) 1 and 3
d) All of the above
Correct Answer: B
Your Answer: Unanswered
Explanation

IASbaba
Web: http://ilp.iasbaba.com/ Score:
Email: ilp@iasbaba.com 0.00 / 160
Page 36
Set 3 Block 2:
Exam Title :
Economics & ...
Email : misrapulkit@yahoo.in
Contact :

Solution (b)

Real Exchange rate : The real exchange rate is the relative price of foreign goods in terms of
domestic goods. It is equal to the nominal exchange rate times the foreign price level divided by
the domestic price level. It measures the international competitiveness of a country in
international trade. When the real exchange rate is equal to one, the two countries are said to
be in purchasing power parity.

It is the purchasing power of a currency relative to another at current exchange rates and
prices. It is the ratio of the number of units of a given country's currency necessary to buy a
market basket of goods in the other country, after acquiring the other country's currency in the
foreign exchange market, to the number of units of the given country's currency that would be
necessary to buy that market basket directly in the given country.

THINK!

• Purchasing Power Parity

Source : NCERT Class 12th - Introductory Macro Economics

QUESTION 30.
Consider the following regarding bank rates of RBI

1. Repo rate is the short term borrowing rate at which RBI borrows money from banks.

2. Reverse repo rate is the short term lending rate at which RBI lends money to banks.

Choose the INCORRECT statement/s

a) 1 only
b) 2 only
c) Both
d) None
Correct Answer: C
Your Answer: Unanswered
Explanation

Solution (c)

REPO RATE also known as the benchmark interest rate is the rate at which the RBI lends
money to the banks for a short term. When the repo rate increases, borrowing from RBI
becomes more expensive. If RBI wants to make it more expensive for the banks to borrow
money, it increases the repo rate similarly, if it wants to make it cheaper for banks to borrow
money it reduces the repo rate.

REVERSE REPO RATE is the short term borrowing rate at which RBI borrows money from
banks. The Reserve bank uses this tool when it feels there is too much money floating in the
banking system. An increase in the reverse repo rate means that the banks will get a higher
rate of interest from RBI. As a result, banks prefer to lend their money to RBI which is always
safe instead of lending it others (people, companies etc) which is always risky.

IASbaba
Web: http://ilp.iasbaba.com/ Score:
Email: ilp@iasbaba.com 0.00 / 160
Page 37
Set 3 Block 2:
Exam Title :
Economics & ...
Email : misrapulkit@yahoo.in
Contact :

Repo Rate signifies the rate at which liquidity is injected in the banking system by RBI, whereas
Reverse Repo rate signifies the rate at which the central bank absorbs liquidity from the banks.

Increase in Repo Rate and Reverse Repo Rate is a symbol of tightening of the policy .

THINK!

• CRR
• SLR.
• MCFR.

Source : NCERT Class 12th - Introductory Macro Economics

QUESTION 31.
Consider the following

1. Inflation is caused due to decrease in the general level of prices.

2. Deflation is causes due to decrease in the rate of inflation.

3. Disinflation is an act of stimulating the economy by increasing money supply.

4. Reflation is caused due to increase in the general price level of goods.

Choose the INCORRECT statements

a) 1,2 and 4
b) 2,3 and 4
c) All of the above
d) None of the above
Correct Answer: C
Your Answer: Unanswered
Explanation

Solution (c)

All are incorrect.

Inflation is the rate at which the general level of prices for goods and services is rising and,
consequently, the purchasing power of currency is falling. Central banks like RBI attempt to
limit inflation, and avoid deflation , in order to keep the economy running smoothly. It is a
sustained increase in the general price level of goods and services in an economy over a period
of time. When the price level rises, each unit of currency buys fewer goods and services;
consequently, inflation reflects a reduction in the purchasing power per unit of
money .Economists generally believe that high rates of inflation and hyperinflation are caused
by an excessive growth of the money supply . The task of keeping the rate of inflation low and
stable is usually given to monetary authorities . Generally, these monetary authorities are the ce
ntral banks (ex. RBI)that control monetary policy through the setting of interest rates , through
open market operations , and through the setting of banking reserve requirements .

Deflation is a decrease in the general price level of goods and services. It is a contraction in
the supply of circulated money within an economy, and therefore the opposite of inflation. In

IASbaba
Web: http://ilp.iasbaba.com/ Score:
Email: ilp@iasbaba.com 0.00 / 160
Page 38
Set 3 Block 2:
Exam Title :
Economics & ...
Email : misrapulkit@yahoo.in
Contact :
times of deflation, the purchasing power of currency and wages are higher than they otherwise
would have been. Economists generally believe that deflation is a problem in a modern economy
because it increases the real value of debt, especially if the deflation is unexpected. Deflation
may also aggravate recessions. Deflation is the natural condition of economies when the supply
of money is fixed, or does not grow as quickly as population and the economy.

Disinflation is a decrease in the rate of inflation – a slowdown in the rate of increase of the
general price level of goods and services in a nation's gross domestic product over time. It is
the opposite of reflation . Disinflation occurs when the increase in the “consumer price level”
slows down from the previous period when the prices were rising. If the inflation rate is not
very high to start with, disinflation can lead to deflation – decreases in the general price level of
goods and services. Although it is used to describe periods of slowing inflation, disinflation
should not be confused with deflation , which can be harmful to the economy.

Reflation is the act of stimulating the economy by increasing the money supply or by reducing
taxes , seeking to bring the economy (specifically price level) back up to the long-term trend,
following a dip in the business cycle . It is fiscal or monetary policy , designed to expand a
country's output and curb the effects of deflation .It is the opposite of disinflation , which seeks
to return the economy back down to the long-term trend. Reflation policies can include
reducing taxes, changing the money supply and lowering interest rates. The term "reflation" is
also used to describe the first phase of economic recovery after a period of contraction.

THINK!

• Stagflation
• Skewflation
• Inflation Targeting

QUESTION 32.
Consider the following statements

1. Excess of total government spending above the national income is known as Deflationary
Gap.
2. Short fall in total spending of the government over the national income is known as
Inflationary Gap.

Choose the correct statement/s

a) 1 only
b) 2 only
c) Both
d) None
Correct Answer: D
Your Answer: Unanswered
Explanation

IASbaba
Web: http://ilp.iasbaba.com/ Score:
Email: ilp@iasbaba.com 0.00 / 160
Page 39
Set 3 Block 2:
Exam Title :
Economics & ...
Email : misrapulkit@yahoo.in
Contact :

Solution (d)

None of them are correct.

Inflationary Gap is defined as excess of total government spending above the national income
(fiscal deficit). It is the amount by which the actual gross domestic product exceeds potential
full-employment GDP. The inflationary gap is always an ex-ante phenomenon, it is always
expected to occur in the future. The concept of the inflationary gap was first given by John
Maynard Keynes in his work How to Pay for War? (1940) this method was basically employed to
study and solve problems regarding war finance. It arises when expected expenditure will not
equal expected consumption at a future date.

Deflationary Gap/Output Gap/GDP Gap is created when there is a shortfall in total spending
of the government (fiscal surplus) over the national income. It is the difference between actual
GDP / actual output and potential GDP . It is a situation of producing more than the demand and
the economy usually heads for a general slowdown in the level of demand.

THINK!

• Recession Gap

QUESTION 33.
Consider the following Non-Banking Financial Companies (NBFC's) and their regulatory bodies

1. Merchant banking companies- SEBI.


2. Nidhi Companies - Ministry of corporate affairs.
3. Housing Finance Companies- Respective state governments.

Choose the correct match

a) 1 and 3
b) 2 and 3
c) 1 and 2
d) All of the above
Correct Answer: C
Your Answer: Unanswered
Explanation

Solution (c)

A Non Banking Financial Company (NBFC) is a company registered under the Companies
Act, 1956 of India , engaged in the business of loans and advances, acquisition of shares, stock,
bonds hire-purchase, insurance business or chit business but does not include any institution
whose principal business includes agriculture, industrial activity or the sale, purchase or
construction of immovable property. The working and operations of NBFCs are regulated by the
Reserve Bank of India (RBI) within the framework of the [[Reserve Bank of India Act, 1934]]

NBFC's cannot perform most of the bank functions. Some differences of NBFC's with respect to
normal banks are

• They cannot accept demand deposits(so can't even issue cheques) but few NBFC's can
accept time deposits and are called as deposit taking NBFC's.

IASbaba
Web: http://ilp.iasbaba.com/ Score:
Email: ilp@iasbaba.com 0.00 / 160
Page 40
Set 3 Block 2:
Exam Title :
Economics & ...
Email : misrapulkit@yahoo.in
Contact :
• Provides Banking services to People without holding a Bank license.
• An NBFC cannot accept Demand Deposits.
• An NBFC is not a part of the payment and settlement system and as such.
• An NBFC cannot issue Cheques drawn on itself.
• Deposit insurance facility of the Deposit Insurance and Credit Guarantee Corporation is
not available for NBFC depositors, unlike banks.
• An NBFC is not required to maintain Reserve Ratios (CRR, SLR etc.)
• An NBFC cannot indulge Primarily in Agricultural, Industrial Activity, Sale-Purchase,
Construction of Immovable Property
• Foreign Investment allowed up to 100%.
• Foreign Investment allowed up to 100%

Types of NBFC's and their corresponding Regulatory bodies are listed below:

Think!

• Development Finance Institutions.


• MUDRA bank.

QUESTION 34.
Which was the first Development Finance Institution( DFI) established in India?

a) Industrial Development Bank of India (IDBI).


b) Industrial Credit and Investment Corporation of India (ICICI).
c) Industrial Reconstruction Corporation of India (IRCI).
d) Industrial Financial Corporation of India (IFCI).
Correct Answer: D
Your Answer: Unanswered
Explanation

Solution (d)

IFCI is the answer.

IFCI, previously Industrial Finance Corporation of India, is an Indian government


owned development bank to cater to the long-term finance needs of the industrial sector. It was
the first development finance institution established by the Indian government
after independence.

Until the establishment of ICICI in 1991, IFCI remained solely responsible for implementation
of the government's industrial policy initiatives.

In 1993 it was reconstituted as a company to impart higher degree of operational flexibility.


IFCI was allowed to access the capital markets directly.

Development Banks- Industrial Sector

IASbaba
Web: http://ilp.iasbaba.com/ Score:
Email: ilp@iasbaba.com 0.00 / 160
Page 41
Set 3 Block 2:
Exam Title :
Economics & ...
Email : misrapulkit@yahoo.in
Contact :

THINK!

• Agricultural development banks.


• Export-Import development banks.
• Housing development banks.

QUESTION 35.
Which is called as umbrella organisation for all retail payments in India?

a) Unified Payment Interface (UPI).


b) Aadhaar Payments Bridge System (APBS).
c) National Payments Corporation of India (NPCI).
d) National Financial Switch (NFS).
Correct Answer: C
Your Answer: Unanswered
Explanation

Solution (c)

NPCI is the answer.

National Payments Corporation of India (NPCI) is the umbrella organisation for all retail
payment systems in India, which aims to allow all Indian citizens to have unrestricted access to
e-payment services. NPCI is a not-for-profit organisation registered under section 8 of the Comp
anies Act 2013 . The organisation is owned by a consortium of major banks, and has been
promoted by the country’s central bank, the Reserve Bank of India . Its recent work of
developing Unified Payments Interface aims to move India to a cashless society with only digital
transactions.

It has successfully completed the development of a domestic card payment network called RuPa
y , reducing the dependency on international card schemes. The RuPay card is now accepted at
all the ATMs , Point-of-Sale terminals and most of the online merchants in the country. More
than 300 cooperative banks and the Regional Rural Banks (RRBs) in the country have also

IASbaba
Web: http://ilp.iasbaba.com/ Score:
Email: ilp@iasbaba.com 0.00 / 160
Page 42
Set 3 Block 2:
Exam Title :
Economics & ...
Email : misrapulkit@yahoo.in
Contact :
issued RuPay ATM cards. A variant of the card called ‘ Kisan Card ’ is now being issued by all
the Public Sector Banks in addition to the mainstream debit card which has been issued by 43
banks. RuPay cards are also issued under the Jan Dhan Yojana scheme. Presently, there are ten
core promoter banks ( State Bank of India , Punjab National Bank , Canara Bank , Bank of
Baroda , Union Bank of India , Bank of India , ICICI Bank , HDFC Bank , Citibank and HSBC ).

The key products of NPCI are:

National Financial Switch (NFS) which connects 1,98,953 ATMs of 449 banks (91 Member
Banks, 358 Sub- Member) .

Immediate Payment Service (IMPS) provided to 84 member banks, with more than 8.49 crore
MMID (Mobile Money Identifier) issued, and crossed 10 million transactions.

National Automated Clearing House (NACH), has close to 400 banks on board. Aadhaar
Payments Bridge System (APBS) has more than 358 banks.

Cheque Truncation System (CTS) has fully migrated in 3 grids-southern, western & northern
grids from MICR centers.

Aadhaar-Enabled Payment System (AEPS) has 36 member banks.

RuPay – Domestic Card Scheme, has issued over 20 crore cards and enabled 10,70,000 PoS
terminals in the country.The newest and most advanced addition to the NPCI revolution is the
Unified Payments Interface (UPI) which was launched on 11 April 2016

IASbaba
Web: http://ilp.iasbaba.com/ Score:
Email: ilp@iasbaba.com 0.00 / 160
Page 43
Set 3 Block 2:
Exam Title :
Economics & ...
Email : misrapulkit@yahoo.in
Contact :

THINK!

• Unified Payment Interface (UPI).


• RuPay.
• National common mobility card.

QUESTION 36.
Consider the following with reference to Non-Performing Assets (NPA's)

1. Doubtful assets are those assets which are Uncollectible


2. Substandard assets are those assets which has remained NPA for a period less than or
equal to 12 months.
3. Loss assets are those assets which have remained in substandard category for a period of
12 months.

Choose the INCORRECT statement/s

a) 1 and 2
b) 2 and 3

IASbaba
Web: http://ilp.iasbaba.com/ Score:
Email: ilp@iasbaba.com 0.00 / 160
Page 44
Set 3 Block 2:
Exam Title :
Economics & ...
Email : misrapulkit@yahoo.in
Contact :

c) 1 and 3
d) None of the above
Correct Answer: C
Your Answer: Unanswered
Explanation

Solution (c)

An asset is called/concluded as Non-performing asset (NPA): when Loan or advance for


which the principal or interest payment remained overdue for a period of more than 90 days in
respect of term loan.

OR

when Loan or advance for which the principal or interest payment remained overdue for two
harvest seasons but for a period of not exceeding two-and-half years in case of agricultural loan

It is defined as a credit facility in respect of which the interest and/or instalment of principal ha
s remained ‘past due’ for a specified period of time.

In simple terms, an asset is tagged as non-performing when it ceases to generate income for the
lender.

An example of NPA:

Suppose State Bank of India (SBI) gives a loan of Rs. 10 crore to a company (Eg: Kingfisher
Airlines). Consider that they agreed upon for an interest rate of say 10% per annum. Now
suppose that initially everything was good and the market forces were working in support to
the airline industry, therefore, Kingfisher was able to service the interest amount. Later, due to
administrative, technical or corporate reasons suppose the company is not able to pay the
interest rates for 90 days. In that case, a loan given to the Kingfisher Airlines is a good case for
the consideration as NPA .

IASbaba
Web: http://ilp.iasbaba.com/ Score:
Email: ilp@iasbaba.com 0.00 / 160
Page 45
Set 3 Block 2:
Exam Title :
Economics & ...
Email : misrapulkit@yahoo.in
Contact :

IASbaba
Web: http://ilp.iasbaba.com/ Score:
Email: ilp@iasbaba.com 0.00 / 160
Page 46
Set 3 Block 2:
Exam Title :
Economics & ...
Email : misrapulkit@yahoo.in
Contact :

Substandard assets: Assets which has remained Nonperforming assets(NPA) for a period less
than or equal to 12 months.

Doubtful assets : An asset would be classified as doubtful if it has remained in the


substandard asset category for a period of 12 months.

Loss assets : As per RBI, "Loss asset is considered uncollectible and of such little value that its
continuance as a bankable asset is not warranted, although there may be some salvage or
recovery value.

THINK!

• Reasons for NPA.


• Managerial measures to combat NPA.
• Debt Recovery Tribunal, ARC's, SARFAESI ACT.

QUESTION 37.
The main endeavour of SARFAESI ACT -2002 is

a) To allow Non-banking financial companies (NBFC's)/banks to convert their loans into equity
stake for managing NPA's.
b) To empower Non-banking financial companies (NBFC's)/banks to attach secured assets of a
loan defaulter and to sale, auction or manage them without court intervention.
c) To allow Asset Reconstruction companies (ARC's) to buy the NPA from Non-banking
financial companies (NBFC's)/Banks at a lower rate to extract profit from them.
d) To identify the inefficiencies and improve the efficiencies of banks.
Correct Answer: B
Your Answer: Unanswered
Explanation

Solution (b)

The Securitisation and Reconstruction of Financial Assets and Enforcement of Security


Interest Act, 2002 (also known as the SARFAESI Act) is an Indian law. It allows banks and
other financial institution to auction residential or commercial properties to recover loans. The
first asset reconstruction company (ARC) of India, ARCIL , was set up under this act. It
empowers Non-banking financial companies (NBFC's)/banks to attach secured assets of a loan
defaulter and to sale, auction or manage them without court intervention.

IASbaba
Web: http://ilp.iasbaba.com/ Score:
Email: ilp@iasbaba.com 0.00 / 160
Page 47
Set 3 Block 2:
Exam Title :
Economics & ...
Email : misrapulkit@yahoo.in
Contact :

Under this act secured creditors (banks or financial institutions) have many rights for
enforcement of security interest under section 13 of SARFAESI Act, 2002. If borrower of
financial assistance makes any default in repayment of loan or any instalment and his account is
classified as Non-performing Asset by secured creditor, then secured creditor may require
before expiry of period of limitation by written notice to the borrower for repayment of due in
full within 60 days by clearly stating amount due and intention for enforcement. Where he does
not discharge dues in full within 60 days, THEN WITHOUT INTERVENTION OF ANY COURT
OR TRIBUNAL Secured creditor may take possession (including sale, lease, assignment) of
secured asset, or take over management of business of borrower or appoint manager for
secured asset or without taking any of these action may also proceed against guarantor or sell
the pledged asset, if any.

The law does not apply to unsecured loans, loans below 1,00,000 or where remaining debt is
below 20% of the original principal. This law allowed the creation of asset reconstruction
companies (ARC) and allowed banks to sell their non-performing assets to ARCs.

Government has amended the SARFAESI Act in August 2016 to empower the ARCs (Asset
Reconstruction Companies), to rejuvenate Debt Recovery Tribunals (DRTs) and to enhance the
effectiveness of asset reconstruction under the new bankruptcy law. The amendment has given
more regulatory powers to the RBI on the working of ARCs. It was also aimed to empower asset
reconstruction and the functioning of DRTs in the context of the newly enacted bankruptcy law.

IASbaba
Web: http://ilp.iasbaba.com/ Score:
Email: ilp@iasbaba.com 0.00 / 160
Page 48
Set 3 Block 2:
Exam Title :
Economics & ...
Email : misrapulkit@yahoo.in
Contact :
As per the amendment, the scope of the registry that contains the central database of all loans
against properties given by all lenders has been widened to include more information. RBI will
get more powers to audit and inspect ARCs and will get the freedom to remove the chairman or
any director. It can also appoint central bank officials into the boards of ARCs.

RBI will get the power to impose penalties on ARCs when the latter doesn’t follow the central
bank’s directives. Similarly, it can regulate the fees charged by ARCs from banks while dealing
with NPAs. The penalty amount has been increased from Rs 5 lakh to Rs 1 crore.

The amendment has brought hire purchase and financial lease under the coverage of the
SARFAESI Act.

Regarding DRTs, the amendment aims to speed up the DRT procedures. Online procedures
including electronic filing of recovery applications, documents and written statements will be
initiated.

The amendments are important for DRTs as they can play an important role under the new
Bankruptcy law. DRTs will be the backbone of the bankruptcy code and deal with all insolvency
proceedings involving individuals. The defaulter has to deposit 50 per cent of the debt due
before filing an appeal at a DRT.

THINK!

• Debt Recovery Tribunal


• Willful Defaulter

QUESTION 38.
Marginal standing facility is defined as

a) Very short term borrowing scheme for scheduled commercial banks.


b) Very long term borrowing scheme for scheduled commercial banks.
c) Very short term borrowing scheme for scheduled cooperative banks.
d) Very long term borrowing scheme for scheduled cooperative banks.
Correct Answer: A
Your Answer: Unanswered
Explanation

Solution (a)

Marginal Standing Facility (MSF is a very short term borrowing scheme for scheduled
commercial banks. Banks may borrow funds through MSF during severe cash shortage or acute
shortage of liquidity. Some of the features of MSF are

• Interest rate is higher than Repo rate


• Collaterals may include Government-Securities in SLR quota.
• Minimum amount is Rs. 1 Crore and then in multiples of Rs. 1 Crore
• Maximum amount is 2.5% of NDTL (changed w.e.f. july 2013)
• Only Scheduled banks can borrow.

Liquidity Adjustment Facility was introduced in 2000. LAF is a facility provided by the
Reserve Bank of India to scheduled commercial banks to avail of liquidity in case of need or to
park excess funds with the RBI on an overnight basis against the collateral of Government

IASbaba
Web: http://ilp.iasbaba.com/ Score:
Email: ilp@iasbaba.com 0.00 / 160
Page 49
Set 3 Block 2:
Exam Title :
Economics & ...
Email : misrapulkit@yahoo.in
Contact :
securities. RBI accept application for a minimum amount of Rs.5 crore and in multiples of Rs. 5
crore thereafter. LAF enables liquidity management on a day-to-day basis. The operations of
LAF are conducted by way of repurchase agreements called Repos & Reverse Repos.

Differences between liquidity adjustment facility (LAF) and Marginal standing facility (MSF).

THINK!

• Monetary tools of RBI.

QUESTION 39.
When the RBI announces lower cash reserve ratio, it means that

a) The RBI will have more money to lend.


b) The commercial banks have more money to lend.
c) The Government of India will have more money to lend.
d) The scheduled commercial banks have less money to lend.
Correct Answer: B
Your Answer: Unanswered
Explanation

Solution (b)

When the RBI announces lower cash reserve ratio, it means that the commercial banks have
more money to lend.

Cash reserve ratio (CRR) means the share of net demand and time liabilities that banks must
maintain as cash with RBI. Under CRR a certain percentage of the total bank deposits has to be
kept in the current account with RBI which means banks do not have access to that much
amount for any economic activity or commercial activity. Banks can’t lend the money to
corporates or individual borrowers, banks can’t use that money for investment purposes. So,
that CRR remains in current account and banks don’t earn anything on that.

IASbaba
Web: http://ilp.iasbaba.com/ Score:
Email: ilp@iasbaba.com 0.00 / 160
Page 50
Set 3 Block 2:
Exam Title :
Economics & ...
Email : misrapulkit@yahoo.in
Contact :

During less CRR, Commercial banks would have access to so much amount for any economic
activity or commercial activity. Banks can hence lend the money to corporates or individual
borrowers, banks can use that money for investment purposes.

IASbaba
Web: http://ilp.iasbaba.com/ Score:
Email: ilp@iasbaba.com 0.00 / 160
Page 51
Set 3 Block 2:
Exam Title :
Economics & ...
Email : misrapulkit@yahoo.in
Contact :

THINK!

• Statutory Liquidity Ratio(SLR).


• Other Monetary tools of RBI

QUESTION 40.
Priority Sector Lending is the guidelines given by

a) Government of India to the banks for providing a specific portion of bank lending to few
specific sectors like software/IT companies, large industries etc.
b) Reserve Bank of India to the banks for providing a specific portion of bank lending to few
specific sectors like software/IT companies , large industries etc.
c) Government of India to the banks for providing a specific portion of bank lending to few
specific sectors like agriculture and allied activities, micro and small enterprises etc
d) Reserve Bank of India to the banks for providing a specific portion of bank lending to few
specific sectors like agriculture and allied activities, micro and small enterprises etc.
Correct Answer: D
Your Answer: Unanswered
Explanation

Solution (d)

Priority Sector Lending is an important role given by the Reserve Bank of India (RBI) to the
banks for providing a specified portion of the bank lending to few specific sectors like
agriculture and allied activities, micro and small enterprises, poor people for housing, students
for education and other low income groups and weaker sections.

Priority Sector refers to those sectors of the economy which may not get timely and adequate cr
edit in the absence of this special dispensation. This is essentially meant for an all round
development of the economy as opposed to focusing only on the financial sector.

Priority sector was first properly defined in 1972, after the National Credit Council emphasiz
ed that there should be a larger involvement of the commercial banks in the priority sector. The
sector was then defined by Dr. K S Krishnaswamy Committee.

The overall objective of priority sector lending programme is to ensure that adequate
institutional credit flows into some of the vulnerable sectors of the economy, which may not be
attractive for the banks from the point of view of profitability.

IASbaba
Web: http://ilp.iasbaba.com/ Score:
Email: ilp@iasbaba.com 0.00 / 160
Page 52
Set 3 Block 2:
Exam Title :
Economics & ...
Email : misrapulkit@yahoo.in
Contact :

CATEGORIES OF PRIORITY SECTOR:

The broad categories of priority sector for all scheduled commercial banks are as under:

(i) Agriculture and Allied Activities (Direct and Indirect finance): Direct finance to agriculture
shall include short, medium and long term loans given for agriculture and allied activities
directly to individual farmers, Self-Help Groups (SHGs) or Joint Liability Groups (JLGs) of
individual farmers without limit and to others (such as corporate, partnership firms and
institutions) up to Rs. 20 lakh, for taking up agriculture/allied activities.

Indirect finance to agriculture shall include loans given for agriculture and allied activities as
specified in Section I, appended.

This distinction between direct and indirect agriculture is dispensed with. Instead, the lending
to agriculture sector has been re-defined to include (i) Farm Credit (which will include short-
term crop loans and medium/long-term credit to farmers) (ii) Agriculture Infrastructure and (iii)
Ancillary Activities,

(ii) Small Scale Industries (Direct and Indirect Finance): Direct finance to small scale industries
(SSI) shall include all loans given to SSI units which are engaged in manufacture, processing or
preservation of goods and whose investment in plant and machinery (original cost) excluding
land and building does not exceed the amounts specified in Section I, appended.

Indirect finance to SSI shall include finance to any person providing inputs to or marketing the
output of artisans, village and cottage industries, hand-looms and to cooperatives of producers
in this sector.

(iii) Small Business / Service Enterprises: shall include small business, retail trade, professional
& self-employed persons, small road & water transport operators and other service enterprises
as per the definition given in Section I and other enterprises that are engaged in providing or

IASbaba
Web: http://ilp.iasbaba.com/ Score:
Email: ilp@iasbaba.com 0.00 / 160
Page 53
Set 3 Block 2:
Exam Title :
Economics & ...
Email : misrapulkit@yahoo.in
Contact :
rendering of services, and whose investment in equipment does not exceed the amount
specified in Section I, appended.

(iv) Micro Credit : Provision of credit and other financial services and products of very small
amounts not exceeding Rs. 50,000 per borrower to the poor in rural, semi-urban and urban
areas, either directly or through a group mechanism, for enabling them to improve their living
standards, will constitute micro credit.

(v) Education loans: Education loans include loans and advances granted to only individuals for
educational purposes up to Rs. 10 lakh for studies in India and Rs. 20 lakh for studies abroad,
and do not include those granted to institutions;

(vi) Housing loans: Loans up to Rs. 28 lakh in metropolitan cities where population is above 10
lakh and Rs. 20 Lakh at other center s for construction/purchase of a dwelling unit per family
provided total cost of the unit in metropolitan centres and at other centres does not exceed Rs.
35 Lacs and Rs. 25 Lacs respectively. (excluding loans granted by banks to their own
employees) and loans given for repairs to the damaged houses of individuals up to Rs.5 lakh in
metropolitan centres and Rs. 2 Lakh at other centres.

(2) Investments by banks in securitised assets, representing loans to agriculture (direct or


indirect), small scale industries (direct or indirect) and housing, shall be eligible for
classification under respective categories of priority sector (direct or indirect) depending on the
underlying assets, provided the securitised assets are originated by banks and financial
institutions and fulfill the Reserve Bank of India guidelines on securitisation. (3) Under Weaker
Sections : Priority sector loans to the following borrowers are considered under Weaker
Sections category:-

(a) Small and marginal farmers;

(b) Artisans, village and cottage industries where individual credit limits do not exceed `50,000;

(c) Beneficiaries of Swarnajayanti Gram Swarozgar Yojana (SGSY), now National Rural
Livelihood Mission (NRLM) ;

(d) Scheduled Castes and Scheduled Tribes;

(e) Beneficiaries of Differential Rate of Interest (DRI) scheme;

(f) Beneficiaries under Swarna Jayanti Shahari Rozgar Yojana (SJSRY);

(g) Beneficiaries under the Scheme for Rehabilitation of Manual Scavengers (SRMS);

(h) Loans to Self Help Groups;

(i) Loans to distressed farmers indebted to non-institutional lenders;

(j) Loans to distressed persons other than farmers not exceeding `50,000 per borrower to
prepay their debt to non-institutional lenders;

(k) Loans to individual women beneficiaries up to `50,000 per borrower. (L) also called or
known as priority sector advancement (PSA);

(m) Account holders under Pradhan Mantri Jan Dhan Yojana (PMJDY)

(n) Renewable energy sector has also been added to priority sector lending in the year 2015.

IASbaba
Web: http://ilp.iasbaba.com/ Score:
Email: ilp@iasbaba.com 0.00 / 160
Page 54
Set 3 Block 2:
Exam Title :
Economics & ...
Email : misrapulkit@yahoo.in
Contact :

THINK!

• Priority Sector Lending Certificates.

QUESTION 41.
Which among the following is/are Financial Regulator in the Indian Financial Market?

a) NABARD
b) SEBI
c) SIDBI
d) All of the above
Correct Answer: B
Your Answer: Unanswered
Explanation

Solution (b)

RBI, SEBI, PMFRDA and IRDA are the four financial regulators in India.

National Bank for Agriculture and Rural Development (NABARD) is an apex development
financial institution in India, headquartered at Mumbai with branches all over India. The Bank
has been entrusted with "matters concerning policy, planning and operations in the field of
credit for agriculture and other economic activities in rural areas in India. NABARD is active in
developing financial inclusion policy and is a member of the Alliance for Financial Inclusion. It
is one of the premier agencies providing developmental credit in rural areas. NABARD is India's
specialised bank for Agriculture and Rural Development in India. International associates of
NABARD include World Bank-affiliated organizations and global developmental agencies
working in the field of agriculture and rural development. These organizations help NABARD by
advising and giving monetary aid for the upliftment of the people in the rural areas and
optimizing the agricultural process.

Securities and Exchange Board of India (SEBI) is the regulator for the securities market in
India. It was established in the year 1988 and given statutory powers on 30 January 1992
through the SEBI Act, 1992. Securities and exchange Board of India (SEBI) was first
established in the year 1988 AQF as a non-statutory body for regulating the, securities market
.It became an autonomous body by The Government of India on 12 April 1992 and given
statutory powers in 1992 with SEBI Act 1992 being passed by the Indian Parliament.

Small Industries Development Bank of India (SIDBI), is an independent financial institution


aimed to aid the growth and development of micro, small and medium-scale enterprises
(MSME) in India. Set up on April 2, 1990 through an act of parliament, it was incorporated
initially as a wholly owned subsidiary of Industrial Development Bank of India. It is the Principal
Financial Institution for the Promotion, Financing and Development of the Micro, Small and
Medium Enterprise (MSME) sector and for Co-ordination of the functions of the institutions
engaged in similar activities. Besides, it has been playing the development role in several ways
such as support to micro-finance institutions for capacity building and on lending. Recently it

IASbaba
Web: http://ilp.iasbaba.com/ Score:
Email: ilp@iasbaba.com 0.00 / 160
Page 55
Set 3 Block 2:
Exam Title :
Economics & ...
Email : misrapulkit@yahoo.in
Contact :
has opened seven branches christened as Micro Finance branches, aimed especially at
dispensing loans up to 5 lakh.

THINK!

• RBI – Reserve Banks of India


• SEBI – Securities and Exchange Board of India
• PFRDA – Pension Fund Regulatory and Development Authority :
• FMC – Forward Markets Commission :
• IRDA – Insurance Regulatory and Development Authority

QUESTION 42.
Consider the following with reference to Financial Sector Legislative Reforms (FSLRC)

1. FSLRC was constituted by Government of India, ministry of Human Resource


Development.
2. FSLRC was asked to comprehensively review and redraw the legislations governing
India's financial system.
3. FSLRC also recommended a draft Indian financial code to replace the bulk of existing RBI
laws.

Choose the INCORRECT statement/s

a) 1 and 2
b) 1 and 3
c) 2 and 3
d) 1 only
Correct Answer: B
Your Answer: Unanswered
Explanation

Solution (b)

The Financial Sector Legislative Reforms Commission (FSLRC) is a body set up by the
Government of India, Ministry of Finance, on 24 March 2011, to review and rewrite the legal-
institutional architecture of the Indian financial sector.

• The FSLRC constituted by the Ministry of Finance in March 2011, was asked to
comprehensively review and redraw the legislations governing India’s financial system.
• According to the FSLRC, the current regulatory architecture is fragmented and is fraught
with regulatory gaps, overlaps, inconsistencies and arbitrage.
• The FSLRC also recommended a draft Indian Financial Code to replace the bulk of the
existing financial laws.

FSLRC –Draft Financial Code:

IASbaba
Web: http://ilp.iasbaba.com/ Score:
Email: ilp@iasbaba.com 0.00 / 160
Page 56
Set 3 Block 2:
Exam Title :
Economics & ...
Email : misrapulkit@yahoo.in
Contact :

The draft Code is a non-sectoral, principles-based law bringing together laws governing
different sectors of the financial system. It addresses nine components, which the FSLRC
believes any financial legal framework should address:

• Consumer protection: Establishes certain basic rights for all financial consumers and
creates a single unified Financial Redressal Agency (FRA) to serve any aggrieved
consumer across sectors.
• Micro-prudential regulation: Five powers for micro-prudential regulation to reduce the
chances of failure: regulation of entry, regulation of risk-taking, regulation of loss
absorption, regulation of governance and management, and monitoring/supervision.
• Resolution: A unified resolution corporation, dealing with various financial firms, should
be created to intervene when a firm is close to failure.
• Capital controls: Suggests RBI control all capital flows as against the current system of
Ministry of Finance (MoF) regulating in-flow and RBI regulating outflow.
• Systemic risk: Establishing the Financial Stability and Development Council (FSDC) as a
statutory agency taking a leadership role in minimizing systemic risk.
• Development and redistribution: Developing market infrastructure and process would be
the responsibility of the regulator while redistribution policies would be under the
purview of the Ministry of Finance.
• Monetary policy: An executive Monetary Policy Committee (MPC) would be established to
decide on how to exercise the RBI’s powers to pursue targets sets by MoF.
• Public debt management: Proposes a single agency to manage government debt.
• Contracts, trading and market abuse: Establishes the legal foundations for contracts,
property and securities markets.

THINK!

• Drawbacks and Criticisms of FSLRC

IASbaba
Web: http://ilp.iasbaba.com/ Score:
Email: ilp@iasbaba.com 0.00 / 160
Page 57
Set 3 Block 2:
Exam Title :
Economics & ...
Email : misrapulkit@yahoo.in
Contact :

QUESTION 43.
The ownership proportion of Government of India (GOI), State Governments, and Sponsor
banks in Regional rural banks (RRB's) are

a) 50, 35, 15 respectively.


b) 40, 30, 30 respectively.
c) 50, 30, 20 respectively.
d) 50, 15, 35 respectively.
Correct Answer: D
Your Answer: Unanswered
Explanation

Solution (d)

Regional Rural Banks (RRB's) are local level banking organizations operating in different
States of India . They have been created with a view to serve primarily the rural areas of India
with basic banking and financial services . However, RRBs may have branches set up for urban
operations and their area of operation may include urban areas too.

The area of operation of RRBs is limited to the area as notified by Government of India covering
one or more districts in the State. RRBs also perform a variety of different functions. RRBs
perform various functions in following heads:

• Providing banking facilities to rural and semi-urban areas.


• Carrying out government operations like disbursement of wages of MGNREGA workers,
distribution of pensions etc.
• Providing Para-Banking facilities like locker facilities, debit and credit cards

The Regional Rural Banks were owned by the Central Government, the State Government and
the Sponsor Bank (there were five commercial banks, Punjab National Bank , State Bank of
India , Syndicate Bank , United Bank of India and UCO Bank , which sponsored the regional
rural banks) who held shares in the ratios as follows Central Government – 50%, State
Government – 15% and Sponsor Banks – 35%

THINK!

• Local Area Banks.


• Commercial Banks
• Bharatiya mahila bank
• Small finance banks

QUESTION 44.
Consider the following statements with respect to Basel Norms

a) It is a set of standards and practises developed for Indian banks to ensure they maintain
adequate capital to withstand periods of economic stress.
b) It is a set of standards and practises developed for Global banks to ensure they maintain
adequate capital to withstand periods of economic stress.

IASbaba
Web: http://ilp.iasbaba.com/ Score:
Email: ilp@iasbaba.com 0.00 / 160
Page 58
Set 3 Block 2:
Exam Title :
Economics & ...
Email : misrapulkit@yahoo.in
Contact :

c) It is a set of standards and practises developed for Asian banks to ensure they maintain
adequate capital to withstand periods of economic stress.
d) It is a set of standards and practises developed for Reserve Bank Of India to ensure they
maintain adequate capital to withstand periods of economic stress
Correct Answer: B
Your Answer: Unanswered
Explanation

Solution (b)

The Basel Accords is a set of standards and practises developed for Global banks to ensure
they maintain adequate capital to withstand periods of economic stress

It refers to the banking supervision Accords (recommendations on banking regulations)— Basel


I , Basel II and Basel III —issued by the Basel Committee on Banking Supervision (BCBS). They
are called the Basel Accords as the BCBS maintains its secretariat at the Bank for International
Settlements in Basel , Switzerland and the committee normally meets there.

It provides recommendations on banking regulations in regards to capital risk , market risk and
operational risk . The purpose of the accords is to ensure that financial institutions have enough
capital on account to meet obligations and absorb unexpected losses.

THINK!

• Bassel accord in India.

QUESTION 45.
What is true about the term Sterilization in economy?

a) It is the fiscal action in which Government of India seeks to limit the effects of inflow and
outflow of capital on the money supply.
b) It is the monetary action in which Government of India seeks to limit the effects of inflow
and outflow of capital on the money supply.
c) It is the fiscal action in which Reserve Bank of India seeks to limit the effects of inflow and
outflow of capital on the money supply.
d) It is the monetary action in which Reserve Bank of India seeks to limit the effects of inflow
and outflow of capital on the money supply.
Correct Answer: D
Your Answer: Unanswered
Explanation

Solution (d)

Sterilizaton is the monetary action in which Reserve Bank of India seeks to limit the effects of
inflow and outflow of capital on the money supply. This can involve open market operations und
ertaken by the central bank whose aim is to neutralize the impact of associated foreign
exchange operations .

Sterilization is most often used in the context of a central bank that takes actions to negate
potentially harmful impacts of capital inflows – such as currency appreciation and inflation –
both of which can reduce export competitiveness. The sterilization process is used to

IASbaba
Web: http://ilp.iasbaba.com/ Score:
Email: ilp@iasbaba.com 0.00 / 160
Page 59
Set 3 Block 2:
Exam Title :
Economics & ...
Email : misrapulkit@yahoo.in
Contact :
manipulate the value of one domestic currency relative to another, and is initiated in the foreign
exchange market .

THINK!

• Fiscal Policy to control inflow and outflow

QUESTION 46.
Liquidity trap is a situation in which

a) Prevailing interest rates are low and saving rates are high.
b) Prevailing interest rates are high and saving rates are low.
c) Prevailing interest rates are low and saving rates are low.
d) Prevailing interest rates are high and saving rates are high
Correct Answer: A
Your Answer: Unanswered
Explanation

Solution (a)

Liquidity trap is a situation in which prevailing interest rates are low and saving rates are
high making monetary policy ineffective. In a liquidity trap, consumers choose to avoid bonds
and keep their funds in savings, because of the prevailing belief that interest rates will soon
rise. Because bonds have an inverse relationship to interest rates, many consumers do not want
to hold an asset with a price that is expected to decline.

As part of the liquidity trap, consumers continue to hold funds in standard deposit accounts,
such as savings and checking accounts, instead of in other investment options, even when the
central banking system attempts to stimulate the economy through the injection of additional
funds. These consumer actions, often spurred by the belief of a negative economic event on the
horizon, causes monetary policy to be generally ineffective.

One marker of a liquidity trap is particularly low interest rates. These low interest rates can
affect bondholder behaviour, along with other concerns regarding the current financial state of
the nation, resulting in the selling of bonds in a way that is harmful to the economy. Further,
additions made to the money supply fail to result in price level changes, as consumer behavior
leans toward saving funds into lower-risk and highly liquid mechanisms. Without changes to
interest rates, consumers are not motivated to invest into other options.

Low interest rates alone do not define a liquidity trap. For the situation to qualify, there has to
be a lack of bondholders wishing to keep their bonds, and a limited supply of investors looking
to purchase them. Instead, the investors are prioritizing strict cash savings over bond
purchasing. If investors are still interested in holding or purchasing bonds at times when
interest rates are low, even approaching the zero limit, the situation does not qualify as a
liquidity trap

THINK!

• Zero Interest rate policy.

IASbaba
Web: http://ilp.iasbaba.com/ Score:
Email: ilp@iasbaba.com 0.00 / 160
Page 60
Set 3 Block 2:
Exam Title :
Economics & ...
Email : misrapulkit@yahoo.in
Contact :

QUESTION 47.
Consider the following related to Net Foreign Factor Income (NFFI),

1. It is the difference between a nations GNP and GDP.


2. It captures the net flow of income payments between domestic economy and foreign
sector.

Choose the correct statement/s

a) 1 only
b) 2 only
c) Both
d) None
Correct Answer: C
Your Answer: Unanswered
Explanation

Solution (c)

The Net foreign factor income (NFFI) is the difference between a nation’s gross national
product (GNP) and gross domestic product (GDP). Net foreign factor income (NFFI) is the differ
ence between the aggregate amount that a country’s citizens and companies earn abroad, and
the aggregate amount that foreign citizens and overseas companies earn in that country.
Mathematically NFFI=GNP-GDP. NFFI captures the net flow of income payments between
domestic economy and foreign sector.

The NFFI level is generally not substantial in most nations, since factor payments earned by
their citizens and those paid to foreigners more or less offset each other. However, the NFFI's
impact may be significant in smaller nations with substantial foreign investment in relation to
their economy and few assets overseas, since their GDP will be quite high compared to GNP.
NFFI may assume increasing importance in a globalized economy , as people and companies
move across international borders more easily than they did in the past.

THINK!

• NNP
• GNP

QUESTION 48.
Capital expenditure of the government consists of

1. Purchase of lands, buildings, machinery etc

IASbaba
Web: http://ilp.iasbaba.com/ Score:
Email: ilp@iasbaba.com 0.00 / 160
Page 61
Set 3 Block 2:
Exam Title :
Economics & ...
Email : misrapulkit@yahoo.in
Contact :
2. Repayment of loan.
3. Grants given to state governments

Choose the correct statement/s

a) 1 and 3
b) 2 and 3
c) 1 and 2
d) All of the above
Correct Answer: C
Your Answer: Unanswered
Explanation

Solution (c)

Capital Expenditure

An expenditure which either creates an asset (e.g., school building) or reduces liability (e.g.,
repayment of loan) is called capital expenditure.

(A) Capital expenditure which leads to creation of assets are (a) expenditure on purchase of
land, buildings, machinery, (b) investment in shares, loans by Central government to state
government, foreign governments and government companies, cash in hand and (c) acquisition
of valuables. Such expenditures are incurred on long period development programmes, real
capital assets and financial assets. This type of expenditure adds to the capital stock of the
economy and raises its capacity to produce more in future.

(B) Repayment of loan is also capital expenditure because it reduces liability. These
expenditures are met out of capital receipts of the government including capital transfers from
rest of the world

Revenue Expenditure:

Simply put, an expenditure which neither creates assets nor reduces liability is called Revenue
Expenditure, e.g., salaries of employees, interest payment on past debt, subsidies, pension, etc.
These are financed out of revenue receipts. Broadly, any expenditure which does not lead to any
creation of assets or reduction in liability is treated as revenue expenditure.

Generally, expenditure incurred on normal running of the government departments and


maintenance of services is treated as revenue expenditure. Examples of revenue expenditure
are salaries of government employees, interest payment on loans taken by the government,
pensions, subsidies, grants, rural development, education and health services, etc.

It is a short period expenditure and recurring in nature which is incurred every year (as against
capital expenditure which is long period expenditure and nonrecurring in nature). The purpose
of such expenditure is not to build up any capital asset, but to ensure normal functioning of
government machinery. Traditionally, all grants given to state governments are treated as
revenue expenditure even though some of the grants may before creation of assets.

Comparison between Revenue Expenditure and Capital Expenditure

IASbaba
Web: http://ilp.iasbaba.com/ Score:
Email: ilp@iasbaba.com 0.00 / 160
Page 62
Set 3 Block 2:
Exam Title :
Economics & ...
Email : misrapulkit@yahoo.in
Contact :

Revenue Expenditure Capital Expenditure

1. It is incurred for
normal running of
1. It is incurred for acquisition
government
of capital assets.
departments and
maintenance.

2. It does not result in 2. It results in creation of


creation of assets. assets.

3. It is recurring in
2. It is non-recurring in
nature and incurred
nature.
regularly.

4. It is short period 4. It is generally a long period


expenditure. expenditure.

5. For example,
expenditure on 5. For example, construction
medicines and salaries of a hospital building is capital
of doctors for rendering expenditure.
services is

THINK!

• Budget of India.
• Components of Annual financial statement(Budget)

QUESTION 49.
Graphical curve which advocates a relationship between inflation and unemployment in an
economy is called

a) Laffer's curve.

IASbaba
Web: http://ilp.iasbaba.com/ Score:
Email: ilp@iasbaba.com 0.00 / 160
Page 63
Set 3 Block 2:
Exam Title :
Economics & ...
Email : misrapulkit@yahoo.in
Contact :

b) Phillips curve.
c) Friedman curve.
d) Rahn curve.
Correct Answer: B
Your Answer: Unanswered
Explanation

Solution (b)

Phillips Curve :

Graphical curve which advocates a relationship between inflation and unemployment in an


economy. It means decreased unemployment, (i.e., increased levels of employment) in an
economy will correlate with higher rates of inflation .. While there is a short run tradeoff
between unemployment and inflation, it has not been observed in the long run. In 1968, Milton
Friedman asserted that the Phillips curve was only applicable in the short-run and that in the
long-run, inflationary policies will not decrease unemployment. [2] [3] Friedman then correctly
predicted that in the 1973–75 recession , both inflation and unemployment would increase . [3]
The long-run Phillips curve is now seen as a vertical line at the natural rate of unemployment,
where the rate of inflation has no effect on unemployment.

Laffers Curve:

IASbaba
Web: http://ilp.iasbaba.com/ Score:
Email: ilp@iasbaba.com 0.00 / 160
Page 64
Set 3 Block 2:
Exam Title :
Economics & ...
Email : misrapulkit@yahoo.in
Contact :

It illustrates a theoretical relationship between rates of taxation and the resulting levels of gove
rnment revenue . It illustrates the concept of taxable income elasticity—i.e., taxable income cha
nges in response to changes in the rate of taxation.

Friedman Curve:

There is no curve as such :)

Rahn Curve:

IASbaba
Web: http://ilp.iasbaba.com/ Score:
Email: ilp@iasbaba.com 0.00 / 160
Page 65
Set 3 Block 2:
Exam Title :
Economics & ...
Email : misrapulkit@yahoo.in
Contact :

The Rahn curve is a graph used to illustrate an economic theory , proposed in 1996 by
American economist Richard W. Rahn , which indicates that there is a level of government
spending that maximises economic growth . The theory is used by classical liberals to argue for
a decrease in overall government spending and taxation .

There are two interlinked aspects of the Perspectives on the appropriate amount of Government
Debt. One is whether government debt is a burden and two is regarding the issue of financing
the debt.

It has often been argued that public ‘debt does not matter because we owe it to ourselves’. This
is because although there is a transfer of resources between generations, purchasing power
remains within the nation. However, any debt that is owed to foreigners involves a burden since
we have to send goods abroad corresponding to the interest payments.

If the government invests in infrastructure, future generations may be better off, provided the
return on such investments is greater than the rate of interest. The actual debt could be paid off
by the growth in output. The debt should not then be considered burdensome. The growth in
debt will have to be judged by the growth of the economy as a whole. Hence Debt is
burdensome if it reduces future growth in output.

QUESTION 50.
Four stages of business cycle in which economies alternate are

a) Depression, Recovery, Boom and Recession


b) Depression, Reflation, Boom and Inflation
c) Low Inflation, Galloping Inflation, Hyper Inflation and Creeping Inflation
d) Depression, Stagflation, Boom and Recession
Correct Answer: A

IASbaba
Web: http://ilp.iasbaba.com/ Score:
Email: ilp@iasbaba.com 0.00 / 160
Page 66
Set 3 Block 2:
Exam Title :
Economics & ...
Email : misrapulkit@yahoo.in
Contact :
Your Answer: Unanswered
Explanation

Solution (a)

Four stages of business cycle in which economies alternate are Depression, Recovery, Boom and
Recession.

Depression :

Depression is a severe and prolonged downturn in economic activity. In economics, a depression


is commonly defined as an extreme recession that lasts two or more years. A depression is
characterized by economic factors such as substantial increases in unemployment, a drop in ava
ilable credit , diminishing output, bankruptcies and sovereign debt defaults, reduced trade and
commerce, and sustained volatility in currency values. In times of depression, consumer
confidence and investments decrease, causing the economy to shutdown

Recovery :

An economic recovery is a period of increasing business activity signaling the end of a recession
. Much like a recession, an economic recovery is not always easy to recognize until at least
several months after it has begun. Economists use a variety of indicators, including gross
domestic product (GDP), inflation, financial markets and unemployment to analyze the state of
the economy and determine whether a recovery is in progress

Boom :

A boom refers to a period of increased commercial activity within either a business, market,
industry or economy as a whole. For an individual company, a boom means rapid and significant
sales growth, while a boom for a country is marked by significant GDP growth. In the stock
market, booms are associated with bull markets, whereas busts are associated with bear
markets .

Recession:

A recession is a significant decline in activity across the economy, lasting longer than a few
months. It is visible in industrial production, employment, real income and wholesale-retail
trade. The technical indicator of a recession is two consecutive quarters of negative economic
growth as measured by a country's gross domestic product (GDP) , although the National
Bureau of Economic Research (NBER) does not necessarily need to see this occur to call a
recession.

THINK!

• Inflation and its types.


• Stagflation,Hyperflation.

IASbaba
Web: http://ilp.iasbaba.com/ Score:
Email: ilp@iasbaba.com 0.00 / 160
Page 67
Set 3 Block 2:
Exam Title :
Economics & ...
Email : misrapulkit@yahoo.in
Contact :

QUESTION 51.
Allowing someone to die by not doing something that would prolong life is called

a) Voluntary euthanasia
b) Passive euthanasia
c) Non-voluntary euthanasia
d) Active euthanasia
Correct Answer: B
Your Answer: Unanswered
Explanation

Solution (b)

Passive Euthanasia: To forgo treatment or life-prolonging medical support

Do You Know?

Coma versus Persistent Vegetative State (PSV)

· A coma is a condition marked by a profoundly debilitating lack of consciousness. Patients in a


coma won’t be able to perceive or respond to external stimuli. While those in a coma may make
involuntary movements, just as those in a persistent vegetative state can, coma patients have
no level of higher consciousness.

· Therein lies the difference between a coma and a persistent vegetative state: In PVS, patients
may still have low levels of consciousness.

Think

· Article 21 of the Indian Constitution

· Reference to Aruna Shaunbag case in passive Euthanasia

· Santhara

· What are the moral dilemmas raised by ‘Euthanasia’?

Source: http://www.thehindu.com/todays-paper/passive-euthanasia-already-a-law-govt/
article19836220.ece

QUESTION 52.
Defence and International Security Partnership (DISP) is often seen in news, related to India
and

a) United Kingdom
b) United States of America
c) France
d) Israel
Correct Answer: A

IASbaba
Web: http://ilp.iasbaba.com/ Score:
Email: ilp@iasbaba.com 0.00 / 160
Page 68
Set 3 Block 2:
Exam Title :
Economics & ...
Email : misrapulkit@yahoo.in
Contact :
Your Answer: Unanswered
Explanation

Solution (a)

The UK and India will elevate their Defence relationship by establishing capability partnerships
in strategic areas, through which they will work together across the spectrum of doctrine,
training and other elements upon which military effectiveness depends. These partnerships will
enable transfer of strategic capability between the two nations, including defence technologies
and manufacturing in areas of mutual interest.

In support of India-UK technology capability partnerships, the two countries will develop
research into new areas and progress their defence science and technology collaborative
projects.

Read more - http://mea.gov.in/outoging-visit-detail.htm?26020/Defence+and


+International+Security+Partnership

Think

· Exercise AJEYA WARRIOR

· Exercise Indradhanush IV

· Joint Navy Exercise Konkan

Source: http://www.thehindu.com/todays-paper/tp-national/india-uk-to-firm-up-
defence-links/article19836159.ece

QUESTION 53.
Consider the following statements about Multi Commodity Exchange of India (MCX)

1. Crude oil is the highest traded commodity on MCX

2. MCX launched India’s first commodity options in gold

Select the correct statements

a) 1 Only
b) 2 Only
c) Both 1 and 2
d) Neither 1 nor 2
Correct Answer: C
Your Answer: Unanswered
Explanation

Solution (c)

Statement 1

· The Securities and Exchange Board of India (SEBI) has allowed exchanges established in GIFT
City to introduce crude oil futures contracts.

IASbaba
Web: http://ilp.iasbaba.com/ Score:
Email: ilp@iasbaba.com 0.00 / 160
Page 69
Set 3 Block 2:
Exam Title :
Economics & ...
Email : misrapulkit@yahoo.in
Contact :

· India INX would offer two contracts — one with Brent crude as the underlying, and the other
with WTI crude. The contract size has been fixed at 100 barrels each.

· Further, DGCX (Dubai Gold and Commodities Exchange) prices would be taken into account
for ascertaining the settlement price of the contracts. DGCX is the biggest derivatives exchange
in the Middle East.

· Trading interest in crude oil can be gauged from the fact that it is the highest traded
commodity on Multi Commodity Exchange of India (MCX), the country’s largest commodity
bourse in terms of market share.

Source: http://www.thehindu.com/business/Industry/gift-city-bourses-to-offer-oil-
contracts/article19835485.ece

Statement 2

· The Multi-Commodity Exchange of India Ltd (MCX) on Tuesday launched India’s first
commodity options in gold, giving stakeholders a new set of financial instruments to hedge their
price risks.

· MCX has currently launched a Gold Option contract, keeping Gold (1 Kg) futures as the
underlying asset, with expiry on November 28, 2017 and January 29, 2018.

· The European-styled Gold options are hedge-friendly and physically settled, which means on
exercise at expiration the options position develops into a corresponding underlying MCX 1 KG
Gold futures position at the strike price of the exercised options.

· The options product is unique and the first of its kind, giving buyers the right to buy or sell the
underlying asset, but no obligation, at a specified price at the expiry.

· Thus, for buyers, the risk is limited only to the premium paid to the option seller (i.e. Option
writer).

Source: http://www.thehindubusinessline.com/markets/commodities/mcx-launches-
indias-first-commodity-options-in-gold/article9912899.ece

QUESTION 54.
Consider the following statements about National Security Guard (NSG)

1. It is a branch of the Indian Army under the authority of the Indian Ministry of Defence.

2. It is a counter-insurgency force made up of soldiers deputed from other parts of the Indian
Army and is currently deployed in the state of Jammu and Kashmir.

Select the correct statements

a) 1 Only
b) 2 Only
c) Both 1 and 2
d) Neither 1 nor 2
Correct Answer: D
Your Answer: Unanswered

IASbaba
Web: http://ilp.iasbaba.com/ Score:
Email: ilp@iasbaba.com 0.00 / 160
Page 70
Set 3 Block 2:
Exam Title :
Economics & ...
Email : misrapulkit@yahoo.in
Contact :
Explanation

Solution (d)

The National Security Guard (NSG) is an Indian special forces unit under the Ministry of Home
Affairs (MHA). It was raised in 1984, following Operation Blue Star and the assassination of
Indira Gandhi, "for combating terrorist activities with a view to protect states against internal
disturbances".

NSG is under the authority of Ministry of Home Affairs (India), However it is not categorised
under the uniform nomenclature of Central Armed Police Forces. It has a special forces
mandate, and its core operational capability is provided by the Special Action Group (SAG)
which is drawn from the Indian Army. The Special Rangers Group (SRG), the police component
of NSG, which also handles VIP security, is composed of personnel on deputation from other
Central Armed Police Forces and State Police Forces.

The NSG personnel are often referred to in the media as Black Cats because of the black dress
and black cat insignia worn on their uniform.

Think

· Paramilitary Forces of India

· Central Armed Police Forces

· Lone-wolf attack

· Operation Chakravyuh

Source: http://indianexpress.com/article/opinion/columns/las-vegas-shooting-
mumbai-2611-indian-police-narendra-modi-new-pattern-of-urban-terror-4884205/

QUESTION 55.
A contest has been launched for designing a common logo and tagline for Geographical
Indications. It was launched by

a) DIPAM
b) CIPAM
c) Ministry of Tourism
d) Ministry of Earth Sciences
Correct Answer: B
Your Answer: Unanswered
Explanation

Solution (b)

A contest has been launched for designing a common logo and tagline for Geographical
Indications to increase awareness about intellectual property rights.

A Geographical Indication (GI) is primarily an agricultural, natural or a manufactured product


(handicrafts and industrial goods) originating from a definite geographical territory.

IASbaba
Web: http://ilp.iasbaba.com/ Score:
Email: ilp@iasbaba.com 0.00 / 160
Page 71
Set 3 Block 2:
Exam Title :
Economics & ...
Email : misrapulkit@yahoo.in
Contact :

Typically, such a name conveys an assurance of quality and distinctiveness, which is essentially
attributable to the place of its origin. Darjeeling Tea, Tirupathi Laddu, Kangra Paintings,
Nagpur Orange and Kashmir Pashmina are among the registered GIs in India.

The Department of Industrial Policy and Promotion (DIPP), under the ministry, “aims to launch a
certifying GI mark/logo that can be used to identify all registered GIs irrespective of the
categories, and a suitable tagline/slogan for promotion of GIs.

The contest was launched by the Cell for IPR Promotions & Management (CIPAM) MyGov.in
website.

CIPAM, which works under the aegis of the DIPP, has also started “Gift a GI” campaign to help
in branding and promotion of registered GI products.

Think

· #LetsTalkIP

Source: http://www.thehindubusinessline.com/news/dipp-launches-logo-contest-for-
geographical-indications/article9914267.ece

QUESTION 56.
Recently , the Union Cabinet gave its approval for implementation of umbrella scheme of
"Modernisation of Police Forces (MPF). Consider the following statements

1. Out of the total outlay, the Centre will provide for 50 per cent under the scheme

2. Scheme of Special Central Assistance (SCA) for 35 worst LWE affected districts has been
introduced to tackle the issue of underdevelopment in these districts

Select the correct statements

a) 1 Only
b) 2 Only
c) Both1 and 2
d) Neither 1 nor 2
Correct Answer: B
Your Answer: Unanswered
Explanation

Solution (b)

The umbrella scheme, Modernisation of Police Forces (MPF), will be implemented between
2017 and 2020.

The financial outlay for the scheme over the three year’s period is Rs.25,060 crore, out of which
the Central Government share will be Rs.18,636 crore and the States’ share will be Rs.6,424
crore.

Salient Features

IASbaba
Web: http://ilp.iasbaba.com/ Score:
Email: ilp@iasbaba.com 0.00 / 160
Page 72
Set 3 Block 2:
Exam Title :
Economics & ...
Email : misrapulkit@yahoo.in
Contact :

· Special provision has been made under the Scheme for internal security, law and order,
women security, availability of modern weapons, mobility of police forces, logistics support,
hiring of helicopters, upgradation of police wireless, National Satellite Network, CCTNS
project, E-prison project etc.

· Under the umbrella scheme, central budget outlay of Rs.10,132 crore has been earmarked for
internal security related expenditure for Jammu & Kashmir, North Eastern States and left wing
extremism affected States.

· Scheme of Special Central Assistance (SCA) for 35 worst LWE affected districts has been
introduced with an outlay of Rs.3,000 crore to tackle the issue of underdevelopment in these
district.

· An outlay of Rs.100 crore has been earmarked in the North Eastern States for police
infrastructure upgradation, training institutes, investigation facilities etc.

· Implementation of this scheme would bolster the Government's ability to address challenges
faced in different theatres such as areas affected by LWE, Jammu and Kashmir and North East
effectively and undertake development interventions which will catalyze in improving the
quality of life in these areas and help combat these challenges effectively at the same time.

· New initiatives are being introduced to provide assistance to States for upgradation of police
infrastructure, forensic science laboratories, institutions and the equipment available with them
to plug critical gaps in the criminal justice system. Police Stations will be integrated to set up a
national data base of crime and criminals’ records. It will be linked with other pillars of criminal
justice system such as ‘prisons, forensic science laboratories and prosecution offices.

· The umbrella scheme also provides for setting up of a State-of Art forensic science laboratory
in Amravati, Andhra Pradesh and upgradation of Sardar Patel Global Centre for Security,
Counter Terrorism and Anti Insurgency in Jaipur and Gujarat Forensic Science University in
Gandhi Nagar.

Source: http://www.thehindu.com/todays-paper/tp-opinion/accountability-not-armour-
plating/article19842135.ece

QUESTION 57.
IMF lowered India’s growth projection in ‘World Economic Outlook, October 2017’. It cited
which of the following reasons?

1. Currency exchange initiative

2. Country-wide Goods and Services Tax

3. Reduced tariff and nontariff barriers to international trade

Select the correct code:

a) 1 and 2
b) 2 and 3
c) 1 and 3
d) All of the above
Correct Answer: A

IASbaba
Web: http://ilp.iasbaba.com/ Score:
Email: ilp@iasbaba.com 0.00 / 160
Page 73
Set 3 Block 2:
Exam Title :
Economics & ...
Email : misrapulkit@yahoo.in
Contact :
Your Answer: Unanswered
Explanation

Solution (a)

India’s economic growth for 2017 and 2018 will be slower than earlier projections, the
International Monitory Fund (IMF) said in its latest World Economic Outlook released. The
report cited “lingering impact” of demonetisation and the Goods and Services Tax for the
expected slowdown during the current and the next year.

The IMF projected India to grow at 6.7% in 2017 and 7.4% in 2018, which are 0.5 and 0.3
percentage points less than the projections earlier this year, respectively.

Source: https://economictimes.indiatimes.com/news/economy/finance/imf-pares-indias-
fy18-growth-forecast-but-sees-it-regaining-fastest-growing-title-in-fy19/
articleshow/61023286.cms

QUESTION 58.
Tentacled Butterfly Ray is found in

a) Baltic Sea
b) Mediterranean Sea
c) Both (a) and (b)
d) Neither (a) nor (b)
Correct Answer: D
Your Answer: Unanswered
Explanation

Solution (d)

Native: Indian Ocean – western; Pacific – western central

Species Range

IASbaba
Web: http://ilp.iasbaba.com/ Score:
Email: ilp@iasbaba.com 0.00 / 160
Page 74
Set 3 Block 2:
Exam Title :
Economics & ...
Email : misrapulkit@yahoo.in
Contact :

Source: http://www.thehindu.com/sci-tech/energy-and-environment/pondicherry-shark-
red-sea-torpedo-and-tentacled-butterfly-ray-may-have-become-extinct-fear-scientists/
article19840728.ece

QUESTION 59.
Consider the following statements about Pondicherry shark

1. It is native to India only

2. It is Critically Endangered in the IUCN List

Select the correct statements

a) 1 Only
b) 2 Only
c) Both 1 and 2
d) Neither 1 nor 2
Correct Answer: B
Your Answer: Unanswered
Explanation

Solution (b)

Native: Indian Ocean – western; Indian Ocean – eastern; Pacific – northwest; Pacific – western
central

Possibly extinct: China; India; Indonesia (Kalimantan); Malaysia; Oman; Pakistan

IASbaba
Web: http://ilp.iasbaba.com/ Score:
Email: ilp@iasbaba.com 0.00 / 160
Page 75
Set 3 Block 2:
Exam Title :
Economics & ...
Email : misrapulkit@yahoo.in
Contact :

Given that it has not be observed in over 20 years, that most known specimens were captured
before 1900, and that its previously known habitat and area of occurrence face expanding
unregulated fisheries, this species is listed as Critically Endangered.

Range of the Pondicherry shark

Source: http://www.thehindu.com/sci-tech/energy-and-environment/pondicherry-shark-
red-sea-torpedo-and-tentacled-butterfly-ray-may-have-become-extinct-fear-scientists/
article19840728.ece

QUESTION 60.
Consider the following statements about single nucleotide polymorphisms (SNPs)

1. They are the most common type of genetic variation among people

2. SNPs can be used to identify ethnicity and skin colour

Select the correct statements

a) 1 Only
b) 2 Only
c) Both 1 and 2
d) Neither 1 nor 2
Correct Answer: C
Your Answer: Unanswered
Explanation

Solution (c)

IASbaba
Web: http://ilp.iasbaba.com/ Score:
Email: ilp@iasbaba.com 0.00 / 160
Page 76
Set 3 Block 2:
Exam Title :
Economics & ...
Email : misrapulkit@yahoo.in
Contact :

Single nucleotide polymorphisms, frequently called SNPs (pronounced “snips”), are the most
common type of genetic variation among people. Each SNP represents a difference in a single
DNA building block, called a nucleotide. For example, a SNP may replace the nucleotide
cytosine (C) with the nucleotide thymine (T) in a certain stretch of DNA.

SNPs occur normally throughout a person’s DNA. They occur once in every 300 nucleotides on
average, which means there are roughly 10 million SNPs in the human genome. Most
commonly, these variations are found in the DNA between genes. They can act as biological
markers, helping scientists locate genes that are associated with disease. When SNPs occur
within a gene or in a regulatory region near a gene, they may play a more direct role in disease
by affecting the gene’s function.

Most SNPs have no effect on health or development. Some of these genetic differences,
however, have proven to be very important in the study of human health. Researchers have
found SNPs that may help predict an individual’s response to certain drugs, susceptibility to
environmental factors such as toxins, and risk of developing particular diseases. SNPs can also
be used to track the inheritance of disease genes within families. Future studies will work to
identify SNPs associated with complex diseases such as heart disease, diabetes, and cancer.

A study of nearly 300 people living in different parts of India found that nine single-base
variants (single-nucleotide polymorphisms or SNPs) account for 31% variation in the color of
the skin.

The single-base variant rs1426654 accounts for 25-38% of skin colour variation between
Africans and Europeans.

Rs1426654 was one of the four SNPs that had maximum effect on skin pigmentation in people
living here.

The gene variants (allele) that give the skin a darker color due to the presence of higher
amount of melanin pigment are found in people living in south India

Being closer to the equator, the darker skin in the south Indian population protects them from
strong UV rays of the Sun.

The darker skin of people in south India was reflected in higher mean melanin index (a
representation of the amount of melanin in the skin) of 48 compared with mean melanin index
of 39 in the case of people in north India.

The population in east and west India has intermediate values (mean MI of 41). The melanin
index did not vary within a given geographical region.

Source: http://www.thehindu.com/sci-tech/science/common-genetic-variants-for-skin-
colour-in-indians-found/article19840725.ece

QUESTION 61.
Marawi , was in news recently. It is located in

a) Philippines
b) Syria
c) Iraq

IASbaba
Web: http://ilp.iasbaba.com/ Score:
Email: ilp@iasbaba.com 0.00 / 160
Page 77
Set 3 Block 2:
Exam Title :
Economics & ...
Email : misrapulkit@yahoo.in
Contact :

d) Laos
Correct Answer: A
Your Answer: Unanswered
Explanation

Solution (a)

Marawi is the capital and the only city in the province of Lanao del Sur on the island of
Mindanao in the Philippines.

The people of Marawi are called the Maranaos and speak the Maranao language. They are
named after Lake Lanao, which is called Meranau in the language, upon whose shores Marawi
lies. The city is also called the "Summer Capital of the South" because of its higher elevation
and cooler climate, a nickname it shares with Malaybalay, which legally holds the title.

An armed conflict against the ISIL-affiliated Maute group was in progress within the city since
May 2017. The city was liberated on Oct 17, 2017

Source: http://www.thehindu.com/news/international/duterte-declares-marawi-
liberated/article19877487.ece

QUESTION 62.
Consider the following statements about Child Marriage Restraint Act 1929

1. It was introduced by Har Bilas Sarda

2. It was applicable only to Hindus across British India

Select the correct statements

a) 1 Only
b) 2 Only
c) Both 1 and 2
d) Neither 1 nor 2
Correct Answer: A
Your Answer: Unanswered
Explanation

Solution (a)

Child Marriage Restraint Act 1929 (Also known as the Sarda Act), passed on 28 September
1929 in the British India Legislature of India, fixed the age of marriage for girls at 14 years and
boys at 18 years which was later amended to 18 for girls and 21 for boys. It is popularly known
as the Sarda Act, after its sponsor Harbilas Sarda. It came into effect six months later on April
1, 1930 and it applies to all of British India, not just to Hindus. It was a result of social reform
movement in India. Despite strong opposition from the British authorities, the legislation was
passed by the British Indian Government which had a majority of Indians.[4] However, it lacked
implementation from the British Indian government, largely due to the fear of British
authorities losing support from their loyal Hindu and Muslim communalist groups.

Think

IASbaba
Web: http://ilp.iasbaba.com/ Score:
Email: ilp@iasbaba.com 0.00 / 160
Page 78
Set 3 Block 2:
Exam Title :
Economics & ...
Email : misrapulkit@yahoo.in
Contact :

· Prohibition of Child Marriage Act, 2006

· POCSO Act

Source: http://www.thehindu.com/todays-paper/tp-opinion/saving-child-brides/
article19849038.ece

QUESTION 63.
Which of the following arguments are most likely to be used to justify protectionism?

1. To protect strategically important industries

2. To protect industries which are still immature

3. To maximise government tax revenue

4. To protect environmental standards

Select the correct statements

a) 1 and 2
b) 1, 2 and 3
c) 2, 3 and 4
d) 1, 3 and 4
Correct Answer: A
Your Answer: Unanswered
Explanation

Solution (a)

Protectionism refers to government actions and policies that restrict or restrain international
trade

Protectionism is often done with the intent of protecting local businesses and jobs from foreign
competition.

The primary objective of protectionism is to make local businesses or industries more


competitive by increasing the price or restricting the quantity of imports entering the country.

Protectionism temporarily creates jobs for domestic workers. The protection of tariffs, quotas or
subsidies allows domestic companies to hire locally.

If a country is trying to grow strong in a new industry, tariffs will protect it from foreign
competitors.

Think

· Globalisation vs Protectionism

· Free trade

· Anti-dumping

IASbaba
Web: http://ilp.iasbaba.com/ Score:
Email: ilp@iasbaba.com 0.00 / 160
Page 79
Set 3 Block 2:
Exam Title :
Economics & ...
Email : misrapulkit@yahoo.in
Contact :

Source: http://www.thehindu.com/todays-paper/tp-business/world-bank-cautions-
against-protectionism/article19849030.ece

QUESTION 64.
Consider the following statements about Sendai Framework for Disaster Risk Reduction

1. It is the first plan to explain, describe and detail the work that is required from all different
sectors and actors to reduce disaster losses.

2. It is a voluntary but non-binding agreement

Select the correct statements

a) 1 Only
b) 2 Only
c) Both 1 and 2
d) Neither 1 nor 2
Correct Answer: B
Your Answer: Unanswered
Explanation

Solution (b)

The Sendai Framework is a 15-year, voluntary, non-binding agreement which recognizes that
the State has the primary role to reduce disaster risk but that responsibility should be shared
with other stakeholders including local government, the private sector and other stakeholders.
It aims for the following outcome:

‘The substantial reduction of disaster risk and losses in lives, livelihoods and health and in the
economic, physical, social, cultural and environmental assets of persons, businesses,
communities and countries.’

The Sendai Framework is the successor instrument to the Hyogo Framework for Action (HFA)
2005-2015: Building the Resilience of Nations and Communities to Disasters.

The Hyogo Framework for Action 2005-2015: Building the Resilience of Nations and
Communities to Disasters (HFA) is the first plan to explain, describe and detail the work that is
required from all different sectors and actors to reduce disaster losses. It was developed and
agreed on with the many partners needed to reduce disaster risk - governments, international
agencies, disaster experts and many others - bringing them into a common system of
coordination. The HFA outlines five priorities for action, and offers guiding principles and
practical means for achieving disaster resilience. Its goal is to substantially reduce disaster
losses by 2015 by building the resilience of nations and communities to disasters. This means
reducing loss of lives and social, economic, and environmental assets when hazards strike.

Source: http://www.thehindu.com/todays-paper/tp-opinion/averting-disaster/
article19849051.ece

QUESTION 65.

IASbaba
Web: http://ilp.iasbaba.com/ Score:
Email: ilp@iasbaba.com 0.00 / 160
Page 80
Set 3 Block 2:
Exam Title :
Economics & ...
Email : misrapulkit@yahoo.in
Contact :

Consider the following statements about International Organization for Migration (IOM)

1. It was initially established as the Intergovernmental Committee for European Migration


(ICEM) to help resettle people displaced by World War II

2. India holds observer status in the IOM

Select the correct statements

a) 1 Only
b) 2 Only
c) Both 1 and 2
d) Neither 1 nor 2
Correct Answer: A
Your Answer: Unanswered
Explanation

Solution (a)

Established in 1951, IOM is the leading inter-governmental organization in the field of


migration and works closely with governmental, intergovernmental and non-governmental
partners. It was initially established in 1951 as the Intergovernmental Committee for European
Migration (ICEM) to help resettle people displaced by World War II.

As of September 2016, it became a related organization of the United Nations.

With 166 member states, a further 8 states holding observer status and offices in over 100
countries, IOM is dedicated to promoting humane and orderly migration for the benefit of all. It
does so by providing services and advice to governments and migrants.

IOM works to help ensure the orderly and humane management of migration, to promote
international cooperation on migration issues, to assist in the search for practical solutions to
migration problems and to provide humanitarian assistance to migrants in need, including
refugees and internally displaced people.

The IOM Constitution recognizes the link between migration and economic, social and cultural
development, as well as to the right of freedom of movement.

IOM works in the four broad areas of migration management:

· Migration and development

· Facilitating migration

· Regulating migration

· Forced migration.

IOM activities that cut across these areas include the promotion of international migration law,
policy debate and guidance, protection of migrants' rights, migration health and the gender
dimension of migration.

India is a member state.

Observer Status: Bahrain,Bhutan,Cuba,Indonesia,Qatar,Russian Federation,San Marino,Saudi


Arabia

IASbaba
Web: http://ilp.iasbaba.com/ Score:
Email: ilp@iasbaba.com 0.00 / 160
Page 81
Set 3 Block 2:
Exam Title :
Economics & ...
Email : misrapulkit@yahoo.in
Contact :

Source: http://www.thehindu.com/todays-paper/sc-asks-centre-to-strike-a-balance-on-
rohingya-issue/article19857521.ece

QUESTION 66.
Justice Verma Committee, 2013 deals with

a) Centre - state relation


b) Direct taxes
c) Child Rights
d) None of the above
Correct Answer: D
Your Answer: Unanswered
Explanation

Solution (d)

In the aftermath of the gang rape in Delhi, Justice Verma was appointed Chairperson of a three-
member commission tasked with reforming and invigorating anti-rape law. His committee
members were Ex-Solicitor General Gopal Subramaniam and Justice (Retd.) Leila Seth. The
Committee was assisted by a team of young lawyers, law students and academics.

The Committee adopted a multidisciplinary approach interpreting its mandate expansively. The
Report deals with sexual crimes at all levels and with the measures needed for prevention as
well as punishment of all offences with sexual overtones that are on affront to human dignity.
This is on the basis that the issue of sexual assault against women is one that goes to the core
of social norms and values. The Report also deals with the construct of gender justice in India
and the various obstructions to this. The Committee's approach is founded on achieving the
guarantee of equality for all in the Constitution of India.

Justice Verma Committee Report Summary - http://www.prsindia.org/parliamenttrack/


report-summaries/justice-verma-committee-report-summary-2628/

Source: http://www.thehindu.com/todays-paper/tp-opinion/the-legal-message/
article19857410.ece

QUESTION 67.
Consider the following statements

1. Zoonoses are infectious diseases of animals that can naturally be transmitted to humans

2. Zoonoses is only caused by virus and bacteria

Select the correct statements

a) 1 Only
b) 2 Only
c) Both 1 and 2
d) Neither 1 nor 2

IASbaba
Web: http://ilp.iasbaba.com/ Score:
Email: ilp@iasbaba.com 0.00 / 160
Page 82
Set 3 Block 2:
Exam Title :
Economics & ...
Email : misrapulkit@yahoo.in
Contact :
Correct Answer: A
Your Answer: Unanswered
Explanation

Solution (a)

Zoonosis is another name for a zoonotic disease. This type of disease passes from an animal or
insect to a human. Some don’t make the animal sick but will sicken a human.

Zoonotic diseases range from minor short-term illness to a major life-changing illness. Certain
ones can even cause death.

The types of zoonosis include those caused by:

· a virus

· bacteria

· fungus

· parasites

Zoonotic diseases spread by mosquitos and ticks are some of the most serious of these diseases.

Zoonoses can be transmitted in various ways:

· through the air

· by eating contaminated meat or produce

· through close contact with an infected animal

· by touching an area or surface that an infected animal touched

· through insect bites like mosquitos or ticks

Many transmissions occur when people hike, bike, boat, or enjoy other activities in the great
outdoors.

Petting zoos are also common places for a zoonotic disease to be transmitted.

Those who live and work on farms are in close contact with many types of livestock. Livestock is
a common carrier of many zoonoses.

Your family pet can carry ticks and fleas indoors that can then move onto you and your family.

Source: http://www.thehindu.com/sci-tech/health/a-plan-to-stamp-out-animal-tb/
article19862749.ece

QUESTION 68.
Farakka Barrage is located in

a) Bangladesh
b) West Bengal

IASbaba
Web: http://ilp.iasbaba.com/ Score:
Email: ilp@iasbaba.com 0.00 / 160
Page 83
Set 3 Block 2:
Exam Title :
Economics & ...
Email : misrapulkit@yahoo.in
Contact :

c) Bihar
d) Uttar Pradesh
Correct Answer: B
Your Answer: Unanswered
Explanation

Solution (b)

Farakka Barrage is a barrage across the Ganges River, located in the Indian state of West
Bengal, roughly 16.5 kilometres (10.3 mi) from the border with Bangladesh near Chapai
Nawabganj District.

QUESTION 69.
Which one of the following is the best description of 'INS Kiltan' that was in the news recently?

a) anti-submarine warfare stealth corvette


b) nuclear -powered submarine
c) torpedo launch and recovery vessel
d) nuclear – powered aircraft carrier
Correct Answer: A
Your Answer: Unanswered
Explanation

Solution (a)

INS Kiltan

About

• Kamorta class Anti-Submarine Warfare (ASW) stealth corvette built under Project 28
• It was built by the Garden Reach Shipbuilders and Engineers, Kolkata
• This ship is unique, as about 81% is built indigenously and is the first built by India that
has a superstructure made up of carbon fibre composite material.
• This makes it a stealth corvette and makes India one among the few nations that have this
technology or this class of ships.
• The Kamorta-class corvettes are a class of anti-submarine warfare corvettes currently in
service with the Indian Navy.
• The ship derives its name from one of the islands in Aminidivi group of the Lakshadweep
and Minicoy group of islands.

Source: http://indianexpress.com/article/what-is/what-is-ins-kiltan-indian-
navy-4892606/

QUESTION 70.
Consider the following statements about North Eastern Council (NEC)

1. It is a statutory body established in 1972 via the North Eastern Council Act, 1971

2. Sikkim was the last state to be added to the council

IASbaba
Web: http://ilp.iasbaba.com/ Score:
Email: ilp@iasbaba.com 0.00 / 160
Page 84
Set 3 Block 2:
Exam Title :
Economics & ...
Email : misrapulkit@yahoo.in
Contact :

Select the correct statements

a) 1 Only
b) 2 Only
c) Both 1 and 2
d) Neither 1 nor 2
Correct Answer: C
Your Answer: Unanswered
Explanation

Solution (c)

• In 1971, the Indian Central government set up the North Eastern Council by North
Eastern Council Act, 1971. The eight States of North East India viz. Arunachal Pradesh,
Assam, Manipur, Meghalaya, Mizoram, Nagaland, Tripura and Sikkim, are members of the
council, with their respective Chief Ministers and Governors representing them.
• Sikkim was added to the council in the year 2002.
• The headquarters of the council is situated in Shillong and functions under the Ministry of
Development of North Eastern Region.
• The Council was initially set up as an advisory body but now sanctioned as a Regional
planning body since 2002.They now discuss any matter in which the North Eastern States
have a common interest and decide the action to be taken on any such matter.
• This was done so as to take care of the economic and social planning of these states, as
well as to provide mediation in the event of inter-State disputes.

Source: http://www.business-standard.com/article/government-press-release/north-
eastern-council-to-be-reoriented-dr-jitendra-singh-117101701103_1.html

QUESTION 71.
Consider the following statements about Abu Dhabi Investment Authority (ADIA)

1. It is the only sovereign wealth fund not signed up to the Santiago Principles

2. It will become the first institutional investor in National Investment and Infrastructure Fund
(NIIF)'s Master Fund and a shareholder in National Investment and Infrastructure Ltd

Select the correct statements

a) Only 1
b) Only 2
c) Both 1 and 2
d) Neither 1 nor 2
Correct Answer: B
Your Answer: Unanswered
Explanation

Solution (b)

• The Abu Dhabi Investment Authority (ADIA) will invest $1billion in the National
Investment and Infrastructure Fund (NIIF)

IASbaba
Web: http://ilp.iasbaba.com/ Score:
Email: ilp@iasbaba.com 0.00 / 160
Page 85
Set 3 Block 2:
Exam Title :
Economics & ...
Email : misrapulkit@yahoo.in
Contact :
• The agreement has been signed between NIIF Master Fund and a wholly owned
subsidiary of Abu Dhabi Investment Authority (ADIA)
• ADIA will become the first institutional investor in NIIF's Master Fund and a shareholder
in National Investment and Infrastructure Ltd

ADIA

• The Abu Dhabi Investment Authority (ADIA) is a sovereign wealth fund owned by Emirate
of Abu Dhabi (in United Arab Emirates) founded for the purpose of investing funds on
behalf of the Government of the Emirate of Abu Dhabi.
• It manages the Emirate’s excess oil reserves.
• The fund is a member of the International Forum of Sovereign Wealth Funds and is
therefore signed up to the Santiago Principles on best practice in managing sovereign
wealth funds.

Source: http://www.livemint.com/Companies/YqtjRoXyALWGlo7Z5zyyXJ/Abu-Dhabi-
Investment-Authority-to-invest-in-NIIFs-Master-Fu.html

QUESTION 72.
Consider the following statements about National e-Governance Services Ltd ( NeSL )

1. It is India’s first information utility (IU) for bankruptcy cases under the Insolvency and
Bankruptcy Code 2016

2. It is solely owned by Securities and Exchange Board of India (SEBI)

Select the correct statements

a) 1 Only
b) 2 Only
c) Both 1 and 2
d) Neither 1 nor 2
Correct Answer: A
Your Answer: Unanswered
Explanation

Solution (a)

National e-Governance Services Ltd. (NeSL)

• NESL is incorporated as a Union Government company and owned by leading public


financial institutions.
• NESL is an information infrastructure institution dealing with critical financial
information projects that aim at better serving the financial sector and its stakeholders,
besides citizen-centric projects
• NeSL is owned by State Bank of India and Life Insurance Corporation Ltd., among others

Objectives

• To collaborate with banks and financial institutions to offer digital services to customers
that cut across different institutions and create a win-win situation for both.

IASbaba
Web: http://ilp.iasbaba.com/ Score:
Email: ilp@iasbaba.com 0.00 / 160
Page 86
Set 3 Block 2:
Exam Title :
Economics & ...
Email : misrapulkit@yahoo.in
Contact :
• To undertake Projects at a national level for optimizing governance services to residents
of India through digital modes, including in financial services.
• To support India’s Financial Inclusion Strategy to make significant changes in rural
economy

Information Utility role

• NESL, as per the IU design under the IBC, proposes to store and deliver data on
borrowings from financial system with due authentication by both banks and borrowers
and associated safeguards.
• NESL will function under regulatory supervision of the Insolvency and Bankruptcy Board
of India and serve the needs of the banking system by providing data to Insolvency
Professionals on fact of borrowing, fact of default and security interest data while
adhering to both networking principles and the highest information security.

Source: http://www.thehindu.com/business/Industry/information-utility-under-the-ibc/
article19866301.ece

QUESTION 73.
Project CHAMAN is associated with

a) Coal Mine Allocation


b) Horticulture
c) Solar Energy
d) Wind Energy
Correct Answer: B
Your Answer: Unanswered
Explanation

Solution (b)

Project CHAMAN

• CHAMAN (Coordinated programme on Horticulture Assessment and Management using


geo informatics)
• It was launched to use geo-spatial applications for the assessment and management of
horticultural crops
• Under the project, the remote sensing technology and sample survey techniques is used
for production forecasting of major horticultural crops in select districts.
• Other components include geospatial applications for horticultural development and
management planning (site suitability, post-harvest infrastructure, crop intensification,
GIS database creation, orchard rejuvenation, aqua-horticulture).
• Besides, detailed scientific field level studies are conducted for developing technology for
crop identification, yield modelling and disease assessment.
• This project is being implemented by Mahalanob is National Crop Forecast Centre
(MNCFC) using remote sensing technology and is likely to be completed in March 2018.

Source: http://pib.nic.in/newsite/PrintRelease.aspx?relid=171750

IASbaba
Web: http://ilp.iasbaba.com/ Score:
Email: ilp@iasbaba.com 0.00 / 160
Page 87
Set 3 Block 2:
Exam Title :
Economics & ...
Email : misrapulkit@yahoo.in
Contact :
QUESTION 74.
Consider the following statements about Footwear Design & Development Institute (FDDI)

1. It is under the aegis of Ministry of Commerce & Industry

2. It has been declared as Indian institutes of eminence

Select the correct statements

a) 1 Only
b) 2 Only
c) Both 1 and 2
d) Neither 1 nor 2
Correct Answer: A
Your Answer: Unanswered
Explanation

Solution (a)

Footwear Design & Development Institute (FDDI)

• FDDI has been declared an ‘Institute of National Importance’ under Ministry of


Commerce & Industry.
• It now has the autonomy to design its courses as per the requirement of the industry, and
award its own degree to the students.
• FDDI can now independently develop and conduct courses leading to graduate and post
graduate degrees, doctoral and post-doctoral courses and research in the areas of
footwear and leather products design and development and allied fields.

FDDI

• FDDI is a premier Institute, serving as a ‘One stop solutions provider’ in footwear, leather
and allied industry.
• Since its inception in 1986, FDDI has been playing a pivotal role in facilitating Indian
industry by bridging skill gaps in the areas of footwear, leather, fashion, retail and
management.
• FDDI has been functioning as an interface between the untapped talent and industry and
its global counterparts, by fulfilling the demand of skilled man power with its specific
curriculum, state of the art laboratories, world class infrastructure and experienced
faculty.

QUESTION 75.
Which of the following is nearest to Vanuatu?

a) India
b) Australia
c) Japan
d) Italy
Correct Answer: B
Your Answer: Unanswered
Explanation

IASbaba
Web: http://ilp.iasbaba.com/ Score:
Email: ilp@iasbaba.com 0.00 / 160
Page 88
Set 3 Block 2:
Exam Title :
Economics & ...
Email : misrapulkit@yahoo.in
Contact :

Solution (b)

It is a Pacific island nation located in the South Pacific Ocean. The archipelago, which is of
volcanic origin, is some 1,750 kilometres (1,090 mi) east of northern Australia, 540 kilometres
(340 mi) northeast of New Caledonia, east of New Guinea, southeast of the Solomon Islands,
and west of Fiji.

Source: http://www.thehindu.com/news/international/dunkirk-style-evacuation-as-
vanuatu-volcano-pollutes-drinking-water/article19780827.ece

IASbaba
Web: http://ilp.iasbaba.com/ Score:
Email: ilp@iasbaba.com 0.00 / 160
Page 89
Set 3 Block 2:
Exam Title :
Economics & ...
Email : misrapulkit@yahoo.in
Contact :

QUESTION 76.
Consider the following statements about ‘Operation Dynamo’

1. It was the evacuation of Allied soldiers during World War II from the beaches of Italy

2. It was implemented by the Royal Navy

Select the correct statements

a) 1 Only
b) 2 Only
c) Both 1 and 2
d) Neither 1 nor 2
Correct Answer: B
Your Answer: Unanswered
Explanation

Solution (b)

Early in the Second World War, in late May 1940, the Allied forces of British, French and
Belgian troops were trapped by the invading German army on the coast of France and Belgium,
in the area around Dunkirk.

The Dunkirk evacuation, code-named Operation Dynamo and also known as the Miracle of
Dunkirk, was the evacuation of Allied soldiers during World War II from the beaches and
harbour of Dunkirk, in the north of France, between 26 May and 4 June 1940.

Operation Dynamo was the rescue operation implemented by the Royal Navy.

Source: http://www.thehindu.com/news/international/dunkirk-style-evacuation-as-
vanuatu-volcano-pollutes-drinking-water/article19780827.ece

QUESTION 77.
Consider the following statements about Indo-Tibetan Border Police (ITBP)

1. It was raised in the wake of the Sino-Indian War of 1962

2. It is under the aegis of Ministry of Defence

Select the correct statements

a) 1 Only
b) 2 Only
c) Both 1 and 2
d) Neither 1 nor 2
Correct Answer: A
Your Answer: Unanswered
Explanation

IASbaba
Web: http://ilp.iasbaba.com/ Score:
Email: ilp@iasbaba.com 0.00 / 160
Page 90
Set 3 Block 2:
Exam Title :
Economics & ...
Email : misrapulkit@yahoo.in
Contact :

Solution (a)

• The Indo-Tibetan Border Police (ITBP) is one of the five Central Armed Police Forces of
India, raised on 24 October 1962, under the CRPF Act, in the wake of the Sino-Indian War
of 1962.
• Presently, ITBP is deployed on border guarding duties from Karakoram Pass in Ladakh to
Jachep La in Arunachal Pradesh covering 3488 km of Indo-China Border and manning
Border Outposts on altitudes ranging from 9000’ to 18700’ in the Western, Middle and
Eastern sectors of the Indo-China Border.
• ITBPF is a specialized mountain force and most of the officers and men are professionally
trained mountaineers and skiers. Being the first responder for natural disaster, ITBPF has
been carrying out numerous rescue and relief operations across the country.
• The force has about 30 border posts above the height of 15,000 feet and some 50 such
posts above 12,000 feet
• It is under the Ministry of Home Affairs

Source: http://www.thehindu.com/news/national/itbp-raises-maiden-mechanised-
column/article19866510.ece

QUESTION 78.
Rafah Crossing Point is the crossing point between

a) Egypt and Gaza Strip


b) Jordan and Syria
c) Kuwait and Iraq
d) Lebanon and Israel
Correct Answer: A
Your Answer: Unanswered
Explanation

Solution (a)

The Rafah Border Crossing or Rafah Crossing Point is the sole crossing point between Egypt
and Gaza Strip. It is located on the Gaza–Egypt border, which was recognized by the 1979
Israel–Egypt Peace Treaty. The original crossing point was named Rafah land port. Only
passage of persons takes place through the Rafah Border Crossing. All traffic of goods is
diverted to the Kerem Shalom border crossing.

Source: http://www.thehindu.com/news/international/fatah-hamas-sign-reconciliation-
accord/article19846720.ece

QUESTION 79.
Recently ‘ MedWatch ’, a mobile health app was launched by

a) Indian Navy
b) Indian Air Force
c) Indian Army

IASbaba
Web: http://ilp.iasbaba.com/ Score:
Email: ilp@iasbaba.com 0.00 / 160
Page 91
Set 3 Block 2:
Exam Title :
Economics & ...
Email : misrapulkit@yahoo.in
Contact :

d) BHEL
Correct Answer: B
Your Answer: Unanswered
Explanation

Solution (b)

MedWatch

• Indian Air Force has launched an innovative mobile health App named `MedWatch'.
• The concept and content of the App is by the Directorate General of Medical Services (Air)
and it has been developed in-house with zero financial outlay by the Directorate of
Information Technology (DIT).
• It comprises a host of features that will provide authentic health information to all IAF
personnel.
• A Reminder Tool to enable timely immunization for the children of all Air Warriors is an
important component of the App. This will directly enable 'Mission Indradhanush'

Source: http://www.business-standard.com/article/news-cm/launch-of-mobile-health-
app-medwatch-by-the-indian-air-force-117101201088_1.html

QUESTION 80.
Consider the following statements about Global Hunger Index (GHI)

1. It is jointly published by the International Food Policy Research Institute (IFPRI) and World
Health Organisation (WHO).

2. According to the recent index, India has a child-wasting prevalence over 20%.

Select the correct statements

a) 1 Only
b) 2 Only
c) Both 1 and 2
d) Neither 1 nor 2
Correct Answer: B
Your Answer: Unanswered
Explanation

Solution (b)

With a global hunger index (GHI) score of 31.4, India is at the high end of the “serious”
category.

India’s poor performance brings to the fore the disturbing reality of the country’s stubbornly
high proportions of malnourished children—more than one-fifth of Indian children under five
weigh too little for their height and over a third are too short for their age.

Data from the report showed that India’s rank (100) was lower than all its neighbours—Nepal
(72), Myanmar (77), Bangladesh (88), Sri Lanka (84) and China (29)—except Pakistan (106).
Even North Korea (93) and Iraq (78) fared better in hunger parameters and GHI rankings.

IASbaba
Web: http://ilp.iasbaba.com/ Score:
Email: ilp@iasbaba.com 0.00 / 160
Page 92
Set 3 Block 2:
Exam Title :
Economics & ...
Email : misrapulkit@yahoo.in
Contact :

The GHI score is a multidimensional index composed of four indicators—proportion of


undernourished in the population, prevalence of child mortality, child stunting, and child
wasting. On the severity scale, a GHI score of less than 10 means “low” prevalence of hunger
while a score of more than 50 implies an “extremely alarming” situation.

On India, the report said that the country’s top 1% own more than 50% of its wealth, India is
the world’s second largest food producer, yet it is also home to the second highest population of
under-nourished in the world.

The 2017 GHI has been jointly published by the International Food Policy Research Institute
(IFPRI), Concern Worldwide, and Welthungerhilfe.

Source: http://www.livemint.com/Politics/JS3kPSIqoUSRt9QC5JbpTI/Indias-hunger-
problem-is-worse-than-North-Koreas-global-h.html

IASbaba
Web: http://ilp.iasbaba.com/ Score:
Email: ilp@iasbaba.com 0.00 / 160
Page 93
Set 3 Block 2:
Exam Title :
Economics & ...
Email : misrapulkit@yahoo.in
Contact :
Review in

IASbaba
Web: http://ilp.iasbaba.com/ Score:
Email: ilp@iasbaba.com 0.00 / 160
Page 94

S-ar putea să vă placă și